Sei sulla pagina 1di 58

AUDITING THEORY TESTBANKS / REVIEWERS

1. f believes that an understanding with the client has not been established, he or she should
ordinarily
a. Perform the audit with increase professional skepticism.
b. Decline to accept or perform the audit.
c. Assess control risk at the maximum level and perform a primarily substantive audit.
d. Modify the scope of the audit to reflect an increased risk of material misstatement due to fraud.

2. An auditor should design the written audit program so that


a. All material transactions will be selected for substantive testing.
b. Substantive tests prior to the balance sheet date will be minimized.
c. The audit procedures selected will achieve specific audit objectives.
d. Each account balance will be tested under either tests of controls or tests of transactions.

3. Which of the following fraudulent activities most likely could be perpetrated due to the lack of
effective internal controls in the revenue cycle?
a. Fictitious transactions may be recorded that cause an understatement of revenues and
overstatement of receivables.
b. Claim received from customers for goods returned may be intentionally recorded in other
customer’s accounts.
c. Authorization of credit memos by personnel who receive cash may permit the misappropriation
of cash.
d. The failure to prepare shipping documents may cause an overstatement of inventory balances.

4. Accepting an engagement to examine an entity’s financial projection most likely would be


appropriate if the projection were to be distributed to
a. All employees who work for the entity.
b. Potential stockholders who request a prospectus or a registration statement.
c. A bank with which the entity is negotiating for a loan.
d. All stockholders of record as of the report date.

5. In assessing control risk for purchases, an auditor vouches a sample of entries in the voucher
register to the supporting documents. Which assertion would this test of controls most likely
support?
a. Completeness c. Valuation or allocation
b. Existence or occurrence d. Rights and obligations

6. Which of the following comparisons would an auditor most likely make in evaluating an entity’s
costs and expenses?
a. The current year’s accounts receivable with the prior year’s accounts receivable.
b. The current year’s payroll expense with the prior year’s payroll expense.
c. The budgeted current year’s sales with the prior year’s sales.
d. The budgeted current year’s warranty expense with the current year’s contingent liabilities.

7. An auditor is planning an audit engagement for a new client in a business that is unfamiliar to the
auditor. Which of the following would be the most useful source of information for the auditor
during the preliminary planning stage, when the auditor is trying to obtain a general understanding
of audit problems that might be encountered?
a. Client manuals of accounts and charts of accounts.
b. Industry Audit Guides.
c. Prior year documentation of the predecessor auditor.
d. Latest annual and interim financial statements issued by the client.

8. An auditor’s report on financial statements prepared in accordance with the financial reporting
provisions of a contract (that is, a special purpose framework) to comply with the provisions of that
contract should include all of the following, except
a. An opinion as to whether the financial statements are presented fairly, in all material respects,
in accordance with the financial reporting provisions of the contract.
b. A statement that indicates the basis of accounting used.
c. An opinion as to whether the basis of accounting used is appropriate under the circumstances.
d. Reference to the note to the financial statements that describes the basis of presentation.

9. An entity’s management is responsible for the preparation and fair presentation of the financial
statements. Its responsibility includes the following, except
a. Designing, implementing, and maintaining internal control relevant to the preparation and
presentation of financial statements.
b. Making accounting estimates that are reasonable in the circumstances.
c. Selecting and applying appropriate accounting policies.
d. Assessing the risks of material misstatement of the financial statements.

10. Which of the following statements best expresses the objective of the traditional audit of financial
statements?
a. To express an opinion on the fairness with which the statements present financial position,
financial performance, and cash flows in accordance with Philippine Financial Reporting
Standards.
b. To express an opinion on the accuracy with which the statements present financial position,
financial performance, and cash flows in accordance with Philippine Financial Reporting
Standards.
c. To make suggestions as to the form or content of the financial statements or to draft them
in whole or in part.
d. To assure adoption of sound accounting policies and the establishment and maintenance of
internal control.

11. Which of the following statements concerning related party transactions is correct?
a. In the absence of evidence to the contrary, related party transactions should be assumed to
be outside the ordinary course of business.
b. The audit procedures directed toward identifying related party transactions should include
considering whether transactions are occurring but are not being given proper accounting
recognition.
c. An auditor should determine whether a particular transaction would have occurred if the
parties had not been related.
d. An auditor should substantiate that related party transactions were consummated on terms
equivalent to those that prevail in arm’s-length transactions.

12. The auditor shall assemble the audit documentation in a/an


a. Working paper c. Audit file
b. Workpaper d. Audit memorandum

13. The auditor is required to complete the administrative process of assembling the final audit file on
a timely basis after the date of the auditor’s report. The time limit within which to complete the
assembly of the audit file is ordinarily
a. Not more than 30 days after the date of the auditor’s report.
b. Not more than 60 days after the date of the auditor’s report.
c. Not more than 90 days after the end of the entity’s reporting period.
d. Not more than 60 days after the date the entity’s financial statements are authorized for
issue.

14. According to Section 9(A) of the IRR, the Commission upon the recommendation of the Board,
shall create an auditing standard setting body to be known as the
a. Auditing and Assurance Standards Council (AASC)
b. Auditing Standards and Practices Council (ASPC)
c. Auditing Standards Board (ASB)
d. Auditing Standards Council (ASC)

15. The following documents shall be submitted by applicants for the CPA licensure examination,
except
a. Certificate of Live Birth in National Statistics Office (NSO) security paper.
b. Marriage contract in NSO security paper for all married applicants.
c. Marriage contract in NSO security paper for married female applicants.
d. Transcript of records with indication therein of date or graduation and Special Order
number unless it is not required.

16. The following documents are to be submitted by a foreign applicant whose letter/document and
the copy of the law on foreign reciprocity of his/her country/state are satisfactory to the BOA,
except
a. Certificate of Religious Affiliation.
b. Original or authenticated copy of transcript of records or equivalent document of the course
for licensure examination where he/she studied, duly authorized or accredited by his/her
country/state.
c. The original or certified copy of any official documents issued by the Bureau of Immigration
and Deportation allowing the applicant to enter and reside in the Philippines.
d. Certificate of Registration or its equivalent stating that the foreign applicant is duly
registered or licensed CPA or its equivalent in his/her country/state.

17. Which part of the Code of Ethics applies to professional accountants in public practice?
a. Part A c. Part C
b. Part B d. Part D

18. Which of the following fundamental ethical principles prohibits association of professional
accountants with reports, returns, communications or other information that is believed to contain
a materially false or misleading statement?
a. Integrity c. Professional competence and due care
b. Objectivity d. Confidentiality

19. Safeguards created by the profession, legislation or regulation include the following, except
a. Continuing professional development requirements.
b. Professional standards.
c. Firm-wide and engagement specific safeguards.
d. Educational, training and experience requirements for entry into the profession.

20. On which of the following safeguards a professional accountant in public practice cannot rely
solely to reduce threats to an acceptable level?
a. Safeguards created by the profession, legislation or regulation.
b. Firm-wide safeguards.
c. Engagement specific safeguards.
d. Safeguards within the client’s systems and procedures.

21. If the fee quoted for a professional service is so low, it may be difficult for the CPA to perform the
engagement in accordance with applicable technical and professional standards for that price.
This situation may create a self-interest threat to
a. Professional competence and due care c. Integrity
b. Objectivity d. Professional behavior

22. Which of the following is not a contingent fee?


a. A fee that is dependent upon the approval of the assurance client’s loan application
b. An audit fee that is based on 5% of the client’s adjusted net income for the current year
c. A fee that is fixed by a court or other public authority
d. An arrangement whereby no fee will be charged unless a specified finding or result is attained

23. A self-interest threat would be created if the firm, or a member of the assurance team, makes a
loan to an assurance client that is not a bank or similar institution, or guarantees such an
assurance client’s borrowing. The self-interest threat created would be so significant that no
safeguard could reduce the threat to an acceptable level unless the loan or guarantee is
a. Made under normal lending terms, procedures and requirements
b. Immaterial to the firm or the member of the assurance team
c. Immaterial to both or the member of the assurance team and the assurance client
d. Made under normal lending terms, procedures and requirements and the loan or guarantee is
immaterial to both the firm or the member of the assurance team and the assurance client

24. The auditor’s risk assessment procedures should always include the following, except
a. Inquiries of management and of others within the entity
b. Analytical procedures
c. Observation and inspection
d. Substantive test procedures and tests of controls

25. The auditor’s risk assessment procedures


a. By themselves, do not provide sufficient appropriate audit evidence on which to base the audit
opinion
b. Should not consider information obtained from the auditor’s previous experience with entity
c. Are designed to detect material misstatement at the assertion level of classes of transactions,
account balances and disclosures
d. Are designed to test the effectiveness of the entity’s controls

26. Which of the following statements concerning audit risk and its components is incorrect?
a. Regardless of the assessed levels of inherent and control risks, the auditor should always
perform some substantive procedures for material account balances and classes of
transactions
b. The higher the assessment of inherent and control risks, the more evidence the auditor should
obtain from the performance of substantive procedures
c. The assessed level of inherent risk need not be considered in determining the nature, timing,
and extent of substantive procedures required to reduce audit risk to an acceptably low level
d. After obtaining an understanding of the accounting and internal control systems, the auditor
should make a preliminary assessment of control risk, at the assertion level, for each material
account balance or class of transactions

27. The following are components of internal control


a. Control activities c. Control environment
b. The entity’s risk assessment process d. Business risk

28. An entity’s internal control system contains manual elements and often contains automated
elements. Manual elements in internal control may be less reliable than automated elements
because
a. Manual control elements can be more easily bypassed, ignored, or overridden and they also
more prone to simple errors and mistakes
b. Manual control elements facilitate the additional analysis of information
c. Consistency of application of manual control elements can always be assumed
d. Manual control elements include reliance on systems or programs that are inaccurately
processing data, processing inaccurate date, or both

29. Which of the following components of an entity’s internal control includes development and use of
training policies that communicate prospective roles and responsibilities to employees?
a. Monitoring of controls c. Control environment
b. Control activities d. Information and communication

30. Control activities relate to the following, except


a. Segregation of duties
b. Performance reviews
c. An internal audit function
d. Authorization

31. Internal control should be designed to provide reasonable assurance that


a. Management’s planning, organizing, and directing processes are properly evaluated
b. Management’s plans have not been circumvented by employee collusion
c. Material errors or fraud will be prevented, or detected and corrected within a timely period by
employees in the course of performing their assigned duties
d. The internal auditing department’s guidance and oversight of management’s performance is
accomplished economically and efficiently

32. An audit is conducted on the premise that management and, where appropriate, those charged
with governance, have acknowledged and understand that they have responsibilities that are
fundamental to the conduct of an audit in accordance with PSAs. Which of t6he following is not
one of those responsibilities?
a. The preparation of financial statements in accordance with relevant pronouncements issued by
the ASCC
b. The establishment and maintenance of an adequate internal control system that is necessary
to enable the preparation of financial statements that are free from material misstatement,
whether due to fraud or error
c. To provide the auditor with access to all information that is relevant to the preparation of the
financial statements such as records, documentation, and other matters
d. To provide the auditor with unrestricted access to persons within the entity from which the
auditor determines it necessary to obtain evidence

33. In performing an audit of financial statements, the auditor should obtain a sufficient knowledge of
a client’s business and industry to
a. Develop an attitude of professional skepticism concerning management’s financial statement
assertions
b. Make constructive suggestions concerning improvements to the client’s internal control
c. Evaluate whether the aggregation of known misstatements causes the financial statements
taken as a whole to be materially misstated
d. Understand the events and transactions that may have an effect on the client’s financial
statements

34. The establishment of an overall audit strategy involves


I. Determining the characteristics of the engagement that define its scope
II. Ascertaining the reporting objectives of the engagement to plan the timing of the audit and the
nature of the communications required
III. Considering the important factors that will determine the focus of the engagement team’s
efforts.
a. I and II only c. I and III only
b. II and III only d. I, II and III

35. Which of the following should be included in the audit plan?


I. The nature, timing and extent of planned risk assessment procedures, as determined under
PAS 315 (Identifying and Assessing the Risks of Material Misstatements trough Understanding
the Entity and its Environment)
II. The nature, timing and extent of planned further audit procedures at the assertion level, as
determined under PSA 330 (The Auditor’s Responses to Assessed Risks)
b. I only c. Both I and II
c. II only d. Neither I nor II

36. Which of the following matters would an auditor most likely consider when establishing the scope
of the audit?
a. The expected audit coverage, including the number and locations of the entity’s components to
be included
b. The entity’s timetable for r4epoting, such as at interim and final stages
c. The discussion with the status of audit work throughout the engagement and the expected
deliverables resulting from the audit procedures
d. Audit areas where there is a higher risk of material misstatement

37. Audit programs are modified to suit the circumstances of particular engagements. A complete
audit progr5am usually should be developed
a. When the engagement letter is prepared
b. After obtaining an understanding of the control environment and control activities components
of the entity’s internal control
c. After the auditor has obtained an understanding of the entity and its environment, including its
internal control and assessed the risks of material misstatement
d. Prior to beginning the actual audit work

38. In connection with the planning phase of an audit engagement, which of the following statements
is always correct?
a. Final staffing decisions must be made prior to completion of the planning stage
b. Observation of inventory count should be performed at year-end
c. A portion of the audit of a continuing audit client can be performed at interim dates
d. An engagement should not be accepted after the client’s financial year-end

39. An adequate system of internal controls is most likely to detect a fraud perpetrated by a
a. Group of employees in collusion
b. Single employee
c. Group of managers in collusion
d. Single manager
40. Controls should be designed to provide reasonable assurance that
a. Organizational objectives and goals will be achieved economically and efficiently
b. Management’s plans have not been circumvented by worker collusion
c. The internal audit activity’s guidance and oversight of management’s performance is
accomplished economically and efficiently
d. Management’s planning, organizing, and directing processes are properly evaluated

41. Criteria that are embodied in laws or regulations, or issued by authorized or recognized bodies of
experts that follow a transparent due process are called
a. Suitable criteria c. Specifically developed criteria
b. Established criteria d. General criteria

42. In an assurance engagement, the outcome of the evaluation or measurement of a subject matter
against criteria is called
a. Subject matter information c. Assurance
b. Subject matter d. Conclusion

43. The following are characteristics of “direct reporting” assurance engagements, except
a. The subject matter information is in the form of an assertion by the responsible party that is
made available to the intended users
b. The subject matter information is provided to the intended users in the assurance report
c. The practitioner either directly performs the evaluation or measurement of the subject matter
or obtains a representation from the responsible party that has performed the evaluation or
measurement
d. The representation of the responsible party that has performed the evaluation or measurement
of the subject matter is not available to the intended users

44. What type of assurance engagement is involved when the practitioner expresses a negative form
of conclusion?
a. Reasonable assurance engagement
b. Negative assurance engagement
c. Assertion-based assurance engagement
d. Limited assurance engagement

45. Assurance engagement risk is the risk


a. That the practitioner expresses an inappropriate conclusion when the subject matter
information is materially misstated
b. Of expressing an inappropriate conclusion when the subject matter information is not
materially misstated
c. Through loss from litigation, adverse publicity, or other events arising in connection with a
subject matter reported on
d. Of expressing an inappropriate conclusion when the subject matter information is either
materially misstated or not materially misstated

46. The following statements relate to the performance of an assurance engagement other than an
audit or review of historical financial information covered by PSAs and PSREs. Which is incorrect?
a. Those persons who are perform the engagement should collectivity possess the necessary
professional competence
b. The practitioner is not allowed to use the work of persons from other professional disciplines
c. The practitioner should consider materiality and assurance engagement risk when planning
and performing an assurance engagement
d. The assurance report should be in writing and should contain a clear expression of the
practitioner’s conclusion about the subject matter information

47. Reducing assurance engagement risk to zero is very rarely attainable or cost beneficial as a result
of the following factors, except
a. The use of selective testing
b. The fact that much of the evidence available assurance knowledge is persuasive rather than
conclusive
c. The practitioner may not have the required assurance knowledge and skills to gather and
evaluate evidence
d. The use of judgment in gathering and evaluating evidence and forming conclusions based on
that evidence

48. The Philippine Standards on Quality Control (PSQCs) are to be applied to


a. Assurance engagements only
b. Review engagements only
c. Compilation and review engagements only
d. All services that fall under the AASC’s engagement standards

49. The overall objectives of the auditor in conducting an audit of financial statements are
I. To obtain reasonable assurance about whether the financial statements as a whole are free
from material misstatements, whether caused by fraud or error
II. To report on the financial statements
III. To obtain conclusive rather than persuasive evidence
IV. To detect all misstatements, whether due to fraud or error
a. I and II only c. I, II and III only
b. II and IV only d. I, II, III and IV

50. The auditor is required to maintain professional skepticism throughout the audit. Which of the
following statements concerning professional skepticism is false?
a. A belief that management and those charged with governance are honest and have integrity
relieves the auditor of the need to maintain professional skepticism
b. Maintaining professional skepticism throughout the audit reduces the risk of using
inappropriate assumptions in determining the nature, timing and extent of the audit procedures
and evaluating the results thereof
c. Professional skepticism is necessary to the critical assessment of audit evidence
d. Professional skepticism is an attitude that includes questioning contradictory audit evidence
obtained

51. Professional judgment


a. Should be exercised in planning and performing an audit of financial statements but need not
be documented
b. Can be used as the justification for the decisions made by the auditor that are not supported by
the facts and circumstances of the engagement
c. Is necessary in the evaluation of management’s judgments in applying the entity’s applicable
financial reporting framework
d. Is not used in making decisions about materiality and audit risk

52. The internal auditing profession has advanced primarily as a result of


a. Increased interest by Bachelor of Science in Accountancy (BSA) graduates and experienced
auditors
b. Job qualification specifications that included added emphasis on background knowledge and
skills
c. The limitation of financial statement audit scope
d. Increased complexity and sophisticated of business operations

53. As defined in PSA 500, _________ is an individual or organization possessing the expertise in a
field other than accounting or auditing, whose work in that field is used by the entity to assist the
entity in preparing the financial statements
a. Auditor’s expert c. Auditor’s internal expert
b. Management’s expert d. Auditor’s external expert

54. Which of the following statements concerning the management’s expert’s competence,
capabilities and objectivity is correct?
a. Objectivity relates to the ability of the management’s expert to exercise the competence in the
circumstances
b. Competence relates to the possible effects that bias, conflict of interest or the influence of
others may have on the professional or business judgment of the management’s expert
c. Capability relates to the nature and level of expertise of the management’s expert
d. The management’s expert’s competence, capabilities and objectivity are important factors in
relation to the reliability of any information prepared by the management’s expert

55. Audit evidence is information used to draw reasonable conclusions on which to base the auditor’s
opinion. Audit evidence is obtained by performing
I. Risk assessment procedures
II. Further audit procedures
a. I only b. II only c. Either I or II d. Both I and II

56. Which of the following would least likely affect the appropriateness of evidence available to an
auditor?
a. The sampling method employed by the auditor to obtain a sample of such evidence
b. The relevance of such evidence to the financial statement assertion being verified
c. The relationship of the preparer of such evidence to the entity being audited
d. The timeless of such evidence

57. The objective of tests of details of transactions performed as substantive tests is to


a. Attain assurance about the reliability of the accounting system
b. Evaluate whether management’s policies and procedures operated effectively
c. Detect material misstatements in the financial statements
d. Comply with generally accepted auditing standards

58. In determining whether transactions have been recorded, the direction of the audit testing should
be from the
a. General journal entries c. General ledger balances
b. Original source documents d. Adjusted trial balance

59. Which of the following elements ultimately determines the specific auditing procedures that are
necessary in the circumstances to afford a reasonable basis for an opinion?
a. Materiality c. Auditor judgment
b. Audit risk d. Reasonable assurance

60. An auditor who uses the work of an expert may refer to and identify the expert in the auditor’s
report if the
a. Expert is employed by the entity
b. Expert’s work provides the auditor greater assurance of reliability
c. Auditor expresses a qualified opinion or an adverse opinion related to the work of the expert
d. Auditor indicates a division of responsibility related to the work of the expert

61.) When an auditor increases the assessed level of control risk because certain control
procedures were determined to be ineffective, the auditor would most likely increase the

a.) Tests of controls b.) Extent of tests of details c.) Tolerable risk d.) Population size

62.) Xavier, CPA is currently auditing the financial statements of the Polk Corporation. At the moment,
Xavier is assessing the control risk surrounding the recognition of sales revenue. The CPA has
become concerned that revenues are understated. The CPA thinks the company may be shipping
merchandise to false customers with no sales invoice (or other record) being prepared. If true,
company employees are using this scheme to steal goods from the company. In assessing the
possibility of this problem, which of the following is the auditor most likely to do?
a) Take a sample of sales invoices and match those documents to the appropriate bill of lading
or other shipping document.
b) Take a sample of receiving reports and match those documents to the appropriate sales
invoice.
c) Take a sample of cash receipts and match those payments to the appropriate sales invoice.
d) Take a sample of bills of lading (or other shipping document) and match those
documents to the appropriate sales invoice.

63.) An auditor is assessing the level of inherent risk in an audit engagement and finds that it is higher
than expected. Which of the following results is most likely?
a) The acceptable level of audit risk should be raised.
b) The desired level of detection risk should be reduced
c) The acceptable level of audit risk should be lowered
d) The desired level of control risk should be reduced.

64.) Epptons, CPA, is auditing the financial statements of a small rural municipality. The receivable
balances represent residents' delinquent real estate taxes. The internal control structure at the
municipality is weak. To determine the existence of the accounts receivable balances at the balance
sheet date, Epptons would most likely
a) Send negative confirmation requests.
b) Send positive confirmation requests.
c) Inspect the internal records such as copies of the tax invoices that were mailed to the
residents.
d) Examine evidence of subsequent cash receipts
**Confirmation of receivables is a generally accepted auditing procedure. The use of positive
confirmation requests is preferable when individual account balances are relatively large or when
there is reason to believe that there may be a substantial number of accounts in dispute or with
inaccuracies or irregularities. The negative form is useful when internal control surrounding accounts
receivable is considered to be effective. In this case, although the individual accounts may not be
relatively large, the internal control structure is weak and the balances, since they are delinquent,
may be in dispute

65.) Which of the following computer-assisted auditing techniques allows fictitious and real
transactions to be processed together without client operating personnel being aware of the testing
process?
a)Parallel simulation.
b)Generalized audit software programming.
c) Test data approach.
d) Integrated test facility.
An integrated test facility is a method of testing programmed controls by creating a small subsystem
within the regular EDP system. Dummy files and records are appended to existing client files and
fictitious test transactions, specifically coded to correspond with the dummy files and records, are
introduced into a system together with actual real transactions.

66.) Tests of controls may include which of the following types of evidence?
a) Inquiry.
b) Observation.
c) Inspection.
d) All of the above.

67.) In accordance with SAS No. 105, which of the following correctly denotes the revised wording of
the first general standard of auditing?
a) The audit must be performed by a person or persons with a college degree in
accounting.
b) The audit must be performed by a person or persons who are partners in a CPA firm.
c) The audit must be performed by a person or persons having adequate technical
training and proficiency as an auditor.
d) The audit should be performed by a person or persons having adequate technical
training and proficiency as an auditor.

68.) When control risk is assessed as low for assertions related to payroll, substantive tests of payroll
balances most likely would be limited to applying analytical procedures and
a) Recalculating payroll accruals.
b) Observing the distribution of paychecks.
c) Footing and cross footing the payroll register.
d) Inspecting payroll tax returns.
**When the auditor determines that the internal control structure is effective and thus the control risk
is assessed as low, the auditor may alter the nature, timing, and extent of substantive tests
performed. In the case of assertions related to payroll, the auditor may decide to limit substantive
tests to performing analytical procedures, which would evaluate the reasonableness of payroll-related
amounts for the year, and recalculating payroll accruals, which would provide some assurance that
the year-end adjustments are proper.

69.) Which of the following procedures would be most effective in reducing attestation risk?
a) Discussion with responsible individuals.
b) Examination of evidence.
c) Inquiries of senior management.
d) Analytical procedures.
**Examination of evidence is the most effective procedure. AT 101.53 states that "…in the hierarchy
of available attest procedures, those that involve search and verification, particularly when using
independent sources outside the entity, are generally more effective in restricting attestation risk than
those involving internal inquiries and comparison of internal information.

70.) Significant deficiencies and material weaknesses in internal control of a public company must be
reported to which of the following?
a) Members of management who are responsible for the related area of the company.
b) Audit Committee of the company's board of directors
c) The Public Company Accounting Oversight Board.
d) None of the above is correct.

71.) An auditor most likely would assess control risk at the maximum if the payroll department
supervisor is responsible for
a) Comparing payroll registers with original batch transmittal data.
b) Authorizing payroll rate changes for all employees.
c) Examining authorization forms for new employees.
d) Hiring all subordinate payroll department employees.
** Authorization and recording are incompatible functions that should be assigned to different
individuals. In this case, authorization of payroll changes, which is typically the responsibility of the
human resources department, should be segregated from recording payroll, which is the
responsibility of the payroll department. If the payroll department supervisor is responsible for these
two functions, the entity's internal control structure may be inadequate to prevent or detect material
misstatements in the payroll area. Thus, the auditor would assess control risk at the maximum.

72.) In the early stages of an audit engagement, the independent CPA must obtain a general
understanding of internal control. Which of the following is not studied as part of that step in the audit
process?
a) Control environment
b) Risk assessment
c) Control activities
d) Internal independence
** The five areas to be studied in gaining a general understanding of internal control are the control
environment, risk assessment, control activities, information and communications, and monitoring.

73.) Which of the following best describes proper internal control over payroll?
a) The payment of cash to employees should be replaced with payment by checks.
b) The confidentiality of employee payroll data should be carefully protected to prevent
fraud.
c) The duties of hiring, payroll computation, and payment to employees should be
segregated.
d) The preparation of the payroll must be under the control of the personnel department.
** The authorization of transactions, record keeping, and custodial functions should be segregated.

74.) The primary purpose of an auditor's consideration of internal control is to provide a basis for
a) Determining whether procedures and records that are concerned with the safeguarding
of assets are reliable.
b) Constructive suggestions to clients concerning improvements in internal control.
c) Determining the nature, extent, and timing of audit tests to be applied.
d) To express an opinion.
** The second standard of fieldwork requires, "A sufficient understanding of internal control is to be
obtained to plan the audit and to determine the nature, timing, and extent of tests to be performed."

75.) Which of the following matters would an auditor most likely consider to be a significant deficiency
to be communicated to management and those in charged with governance?
a) Management's failure to renegotiate unfavorable long-term purchase commitments.
b) Recurring operating losses that may indicate going concern problems.
c) Evidence of a lack of objectivity by those responsible for accounting decisions.
d) Management's current plans to reduce its ownership equity in the entity.
** A significant deficiency is a control deficiency or combination of control deficiencies that adversely
affect the entity's ability to summarize, authorize, record, process or report financial data in
accordance with GAAP, such that a material misstatement of the entity's financial statements will not
be prevented or detected. Thus, a lack of objectivity by those responsible for accounting decisions
would affect management's ability to produce financial statement free from material errors.

76.) In an audit of financial statements in accordance with generally accepted auditing standards, an
auditor is required to
a) Determine whether control procedures are suitably designed to prevent or detect
material misstatements.
b) Perform tests of controls to evaluate the effectiveness of the entity's accounting system.
c) Search for significant deficiencies in the operation of the internal control structure.
d) Document the auditor's understanding of the entity's internal control structure
** The auditor is required to document the auditor's understanding of the entity's internal control
structure. The form and extent of documentation may be influenced by the size and complexity of the
entity, but in all audits this understanding must be documented.

77.) A CPA is performing an audit examination of Malfoy, Inc. The CPA has already done the work
necessary to gain a general understanding of the company’s internal control. The CPA is now looking
at the internal control policies and procedures within the accounting system specifically for accounts
receivable. The CPA has decided to perform test of controls in this area. Which of the following is
least likely to be true?
a) The system contains an excess number of possible problems so that testing is
required.
b) The internal control system appears to be well designed.
c) The CPA hopes to reduce the necessary amount of substantive testing.
d) The CPA hopes to reduce overall audit time.
** Once a general understanding of internal control has been obtained, the auditor only needs to test
specific controls if it is efficient to do so. If the general internal control system appears to be well
designed, testing specific controls to verify their efficiency can lead to a reduction in the assessed
level of control risk. Hopefully, the amount of substantive testing can then be reduced because the
desired level of detection risk does not need to be so low. This decrease in substantive testing can
lead, possibly, to an overall reduction in the time and cost of performing the audit. Conversely, if the
internal control appears to be weak, no benefit is gained by testing those controls further. It is not
likely that those weaknesses can be overcome.

78.) Mr. A makes a 3,000 payment on Monday to Acme Company. The money is stolen by one of
Acme’s employees. On Friday, Ms. B makes a 3,000 payment to Acme Company. The same
employee credits this payment to Mr. A’s account (rather than to Ms. B’s account) so that a second bill
will not be sent to him. Several days later, a payment is received from Mr. C and it is posted to Ms.
B’s account. What is this type of theft known as?
a) Posting Fraud
b) Reconciliation Shortage
c) Lapping
d) Receivable Management Deficiency (RMD)
** The term “lapping” has long referred to the coverage of theft by using amounts received from one
customer to cover earlier balances received from other customers and then stolen. If lapping is not
performed, the original customer will receive a second invoice and any subsequent complaint by the
customer will alert management to the problem.

79.) Effective internal control over purchases generally can be achieved in a well-planned
organizational structure with a separate purchasing department that has
a) The authority to make purchases of requisitioned materials and services.
b) A direct reporting responsibility to the controller of the organization.
c) The responsibility of reviewing purchase orders issued by user departments
d) The ability to prepare payment vouchers based on the information on a vendor's invoice.
** Purchasing agent has authority to purchase, not authority to initiate purchases.

80.) An auditor uses the assessed level of control risk to


a) Determine the acceptable level of detection risk for financial statement assertions.
b) Evaluate the effectiveness of the entity's internal control policies and procedures.
c) Identify transactions and account balances where inherent risk is at the maximum.
d) Indicate whether materiality thresholds for planning and evaluation purposes are
sufficiently high.
** According to the audit risk model, acceptable detection risk is a function of allowable audit risk,
inherent risk, and control risk. This question is concerned with control risk, which is defined as the risk
that a material misstatement that could occur in an assertion will not be prevented or detected on a
timely basis by an entity's internal control structure policies or procedures, and detection risk, which is
defined as the risk that the auditor will not detect a material misstatement that exists in an assertion.
The auditor uses the assessed level of control risk to determine the acceptable level of detection risk
for financial statement assertions.

81.) In reporting on an entity's internal control structure over financial reporting, a practitioner should
include a paragraph that describes the
a) Inherent limitations of any internal control structure.
b) Documentary evidence regarding the control environment factors.
c) Changes in the internal control structure since the prior report.
d) Potential benefits from the practitioner's suggested improvements.
** A practitioner's report on an examination of management's assertion about the effectiveness of the
entity's internal control structure should include a paragraph stating that, because of inherent
limitations of any internal control structure, errors or irregularities may occur and not be detected.

82.) An auditor uses the knowledge provided by the understanding of internal control and the
assessed level of control risk primarily to
a) Determine whether procedures and records concerning the safeguarding of assets are
reliable.
b) Determine the nature, timing, and extent of substantive tests for financial
statement assertions.
c) Modify the initial assessments of inherent risk and preliminary judgments about
materiality levels.
d) Ascertain whether the opportunities to allow any person to both perpetrate and conceal
irregularities are minimized.
** The second standard of field work states that, "The auditor should obtain a sufficient
understanding of the internal control structure to plan the audit and to determine the nature, timing
and extent of tests to be performed." These tests are the substantive tests for financial statement
assertions

83.) The auditor should use the risk assessment to determine:


a) Whether to accept the engagement.
b) The type of opinion to issue in the Audit Report.
c) The size of the audit team.
d) The nature, timing, and extent of further audit procedures to be performed.

84.) After obtaining an understanding of an entity's internal control and assessing control risk, an
auditor may next
a) Perform tests of controls to verify management's assertions that are embodied in the
financial statements.
b) Apply analytical procedures as substantive tests to validate the assessed level of control
risk.
c) Evaluate whether the internal control structure policies and procedures detected material
misstatements in the financial statements.
d) Consider whether evidential matter is available to support a further reduction in
the assessed level of control risk.
** After the auditor assesses control risk, the auditor may desire a further reduction in the assessed
level of control risk for some assertions. The auditor would then decide if it is likely that additional
evidential matter could be obtained to support a lower assessed level of control risk for these
assertions. If yes, and it is likely to be efficient to obtain such evidential matter, the auditor would then
perform additional tests of controls. Next, whether the auditor performed additional tests of controls or
not, the auditor would document the basis for conclusions about the assessed level of control risk and
design substantive tests.

85.) Which of the following is not a component of an entity's internal control?


a) The control environment.
b) Risk assessment.
c) Information and communication.
d) Control risk.

86.) Which of the following is responsible for establishing internal controls for a public company?
a) Management.
b) Management and auditors.
c) Committee on Sponsoring Organizations.
d) Financial statement auditors.
87.) The auditor's study of a public company's internal control is:
a) Recommended by the PICPA.
b) Required by GAAS.
c) Required by PICAP.
d) Required by the Sarbanes-Oxley Act.
88.) The auditor's study of a private company's internal control is:
e) Recommended by the PICPA.
f) Required by GAAS.
g) Required by PICAP.
h) Required by the Sarbanes-Oxley Act.
89.) Which of management's concerns with respect to implementing internal controls is the auditor
primarily concerned?
a) Reliability of financial reporting.
b) Efficiency of operations.
c) Compliance wit applicable laws and regulations.
d) Effectiveness of operations.
90.) When an auditor attempts to understand the operation of the accounting system by tracing a few
transactions through the accounting system, the auditor is said to be:
a) Testing controls.
b) Vouching.
c) Tracing.
d) Performing a walk-through.
** In a walk-through, the auditor selects one or a few documents of a transaction type and traces
them from initiation through the entire accounting process. Walk-through conveniently combines
observation, documentation, and inquiry.
91.) The primary emphasis by auditors by auditor when assessing internal control is on controls over:
a) Account balances.
b) Classes of transactions.
c) Both A and B, because they are equally important.
d) Both A and B, because they vary from client to client.
** Auditors emphasize internal control over classes of transactions rather than account balances
because the accuracy of accounting system outputs (account balances) depends heavily on the
accuracy of inputs and processing (transactions).

92.) Narrative, flowcharts, and internal control questionnaires are three common methods of:
a) Documenting the auditor's understanding of internal controls.
b) Designing the audit manual and procedures.
c) Testing the internal controls.
d) Documenting the auditor's understanding of the client's organizational structure.

93.) In obtaining an understanding of a manufacturing entity's internal control concerning inventory


balances, an auditor most likely would
a) Perform test counts of inventory during the entity's physical count.
b) Analyze inventory turnover statistics to identify slow-moving and obsolete items.
c) Review the entity's descriptions of inventory policies and procedures.
d) Analyze monthly production reports to identify variances and unusual transactions.
** An auditor is required to obtain an understanding of a client's internal control structure. Reviewing
policies and procedures manuals that describe a client system such as inventory and the related
controls is a standard audit step in obtaining that understanding.

94.) The auditor who becomes aware of a reportable condition in internal control is required to
communicate this to the
a) Senior management and board of directors.
b) Board of directors and internal auditors.
c) Internal auditors and senior management.
d) Audit committee or its equivalent.
** The audit committee is the appropriate recipient of communication regarding internal control related
matters.
95.) After obtaining an understanding of internal control and assessing control risk, an auditor decide
to perform tests of controls. The auditor most likely decided that
a) It would be efficient to perform tests of controls that would result in a reduction in
planned substantive tests.
b) There were many internal control structure weaknesses that could allow errors in the
accounting systems.
c) Additional evidence to support a further reduction in control risk is not available.
d) An increase in the assessed level of control risk is justified for certain financial statement
assertions.
** Auditing authoritative sources state that after obtaining an understanding of internal control, the
auditor considers if it is sufficient to perform tests of controls that would result in a reduction in
planned substantive tests.

96.) When management is evaluating the design of internal control, management evaluates whether
the control can do all but which of the following?
a) Correct material misstatements.
b) Prevent material misstatements.
c) Detect material misstatements.
d) None of the above is correct.
**Controls are designed to prevent and detect errors; corrections of errors involve human
intervention.

97.) Which of the following may increase risk to an organization?


a) Quality of personnel.
b) Presence of new information technologies.
c) Geographic dispersion of the company operations.
d) All of the above.

98.) Risk assessment for financial reporting is management’s process for identifying, analyzing, and
responding to risks relevant to the preparation of financial statements in conformity with:
a) Generally Accepted Accounting Standards
b) Generally Accepted Auditing Standards.
c) PCAOB Auditing Standards.
d) Generally Accepted Accounting Principles

99.) An auditor is currently assessing control risk and finds that it is lower than had been anticipated.
How does this discovery impact the work of the auditor?
a) The acceptable level of audit risk can be raised.
b) The acceptable level of inherent risk can be lowered.
c) The acceptable level of audit risk can be lowered.
d) The acceptable level of detection risk can be raised.

100.) Risk assessment involves considering threats to the organization’s objectives in the areas of:
a) Marketing, financial reporting and compliance.
b) Operations, financial reporting and compliance with laws and regulations.
c) Financial reporting, performance and marketing.
d) Compliance with laws and regulations, operations and performance.
101.) To obtain an understanding of the entity and its environment, including its internal control, the
auditor should perform each of the following risk assessment procedures except:
a) Inquiries of management and others within the entity.
b) Analytical procedures.
c) Attribute Sampling.
d) Observation and inspection.

102.) In assessing control risk for purchases, an auditor vouches a sample of entries in the voucher
register to the supporting documents. Which assertion would this test of controls most likely support?
a) Valuation or allocation.
b) Rights and Obligations
c) Existence or occurrence.
d) Completeness.
**Assertions about existence or occurrence deal with whether assets or liabilities of the entity exist at
a given date and whether recorded transactions have occurred during a given period. For example,
management asserts that finished goods inventories in the balance sheet are available for sale, or
management asserts that sales in the income statement represent the exchange of goods or services
with customers for cash or other consideration.

103.) The ultimate purpose of assessing control risk is to contribute to the auditor's evaluation of the
risk that
a) Material misstatements may exist in the financial statements.
b) Entity policies may be overridden by senior management.
c) Specified controls requiring segregation of duties may be circumvented by collusion.
d) Tests of controls may fail to identify procedures relevant to assertions.
**Control risk is defined as the risk that a material misstatement that could occur in an assertion will
not be prevented or detected on a timely basis by an entity's internal control structure policies or
procedures. The auditor assesses control risk as part of his overall evaluation of the risk that material
misstatements may exist in the financial statements.

104.) An auditor assesses inherent risk in an engagement. An auditor assesses control risk in an
engagement. Then, based on those two assessments, the auditor does enough substantive testing to
reduce detection risk so that overall audit risk drops to an acceptably low level. That is the structure of
an audit. In assessing control risk, the auditor starts by learning the design of the control system and
then considers whether to perform tests of individual controls. If the design of the control system is
viewed as weak, which of the following is most likely to be true?
a) The auditor will be likely to test the controls because of the weakness.
b) The auditor is likely to do additional substantive testing.
c) The auditor will likely reduce the assessment of inherent risk.
d) An unqualified opinion cannot be issued.
**When the design of internal control is weak, there is no reason to do any testing of those controls.
Properly operating within a weak system does not make it any less weak. Instead, the auditor will
probably assess control risk as high which is likely to force the auditor to do additional (or better)
substantive testing in order to compensate for this problem. Detection risk has to be quite low to
make up for the weakness in internal control. The assessment of inherent risk is independent from
the assessment of control risk and is not affected. Finally, as long as the auditor eventually reduces
overall audit risk to an appropriately low level, an unqualified opinion can be given even though the
control system is weak.

105.) An independent CPA is assessing the level of control risk present at a company that is currently
being audited. This company has an internal audit department and the independent CPA is evaluating
the objectivity of that department. Which of the following is a sign that the department has proper
objectivity?
a) The size of the department has grown from 7 people to 19 over the last three years.
b) In order to be promoted within this department, the employee must have become a CPA.
c) The head of the department meets privately with the chair of the board of directors
each quarter.
d) The head of the department had seven years of experience with an international
accounting firm before taking this position with the company.
**The independent CPA is interested in both the competence and objectivity of the internal audit
department because of the key function that it plays in the company. Answers A, B, and D all relate to
competence. These relate to the question: Is there evidence that the people doing the work are
capable of doing it properly? Objectivity concerns the internal auditor’s independence so that the
department can do the work that is necessary. That is usually shown by having a direct line of
communication to an authority figure outside of management (usually the board of directors or the
audit committee of the board of directors).

106.) One of the components of internal control that an independent auditor must come to understand
about each audit client is “information and communication.” What is meant by this term?
a) The ability of the management of the company to communicate its priorities to the
appropriate staff levels within the organization.
b) The ability of the accounting system to generate reliable information and convey it
in a timely manner to those parties within the organization that needs it.
c) The ability of employees in a company to warn the independent auditor of fraudulent
actions within the organization.
d) The ability of the internal auditor to communicate information about the various systems
to people within the organization at an appropriate level of authority.
**Internal control includes any policies and procedures within the company designed to ensure that
the accounting systems are functioning effectively as designed by the management of the company.
One general goal is to make sure that the information produced by the accounting system is reliable
and reaches the correct party in time to be used in making appropriate decisions. In looking at a
particular system and its internal control, the auditor evaluates the ability to generate information and
then communicate it to the parties who can make use of it.

107.) A CPA firm is beginning the audit of Panasian Corporation. One of the staff auditors has been
assigned to gain and then document her understanding of the internal controls designed to be in
place in the company’s payroll system. At the end of the day, the staff auditor has created a series of
flowcharts, questionnaires, and narrative descriptions based on the understanding she has obtained.
Which of the following is correct?
a) The questionnaire approach is preferred.
b) The flowchart approach is preferred.
c) She was correct in using all three of these techniques to fulfill this assignment.
d) She only needed to use one of these techniques.
**The auditor’s goal was to establish her understanding of the design of the controls that were
supposed to be in place in this payroll system. All three of these techniques (questionnaire,
flowchart, or narrative) can accomplish this purpose successfully. Therefore, only one is necessary
although sometimes the techniques are grouped together if the system is particularly complex.

108.) Jane Simpson works for Shadowline Corporation and is paid $30 per hour. She typically works
28 hours per week but always reports that she works 32 hours per week so that she can receive
additional wages from the company. Which of the following internal control activities is most likely to
prevent this type of theft?
a) The company’s payroll program is tested each month with test data to ensure that it
operates properly.
b) A separate paymaster delivers the checks each pay period to Jane Simpson after
verifying her identity.
c) The supervisor for Jane Simpson must review her time sheet each period and
indicate approval.
d) Any paychecks that are printed but not picked up must be turned over to an independent
group for subsequent handling.
**Each of these four is an internal control activity frequently found in a company’s payroll system.
However, they are each designed to prevent or discover frauds of a specific type. Here, the problem
is extra hours claimed by an employee so that unearned money can be received. Test data is used
to verify that the payroll program is working as intended. The paymaster hands out checks to make
certain that checks are being prepared for actual individuals who work for the company. That is also
the case for following up on checks that are not claimed at the appropriate time. The approval by the
supervisor is correct here because the supervisor is the person most likely to know how many hours
the employee actually did work. That person is in a position to verify that the number of hours listed
is correct.

109.) Which of the following audit procedures would an auditor most likely perform to test controls
relating to management's assertion concerning the completeness of sales transactions?

a) Inspect the entity's reports of prenumbered shipping documents that have not
been recorded in the sales journal.
b) Inquire about the entity's credit granting policies and the consistent application of credit
checks.
c) Compare the invoiced prices on prenumbered sales invoices to the entity's authorized
price list.
d) Verify that extensions and footings on the entity's sales invoices and monthly customer
statements have been recomputed.
110.) An auditor uses the assessed level of control risk to
a) Determine the acceptable level of detection risk for financial statement assertions.
b) Evaluate the effectiveness of the entity's internal control policies and procedures.
c) Identify transactions and account balances where inherent risk is at the maximum.
d) Indicate whether materiality thresholds for planning and evaluation purposes are
sufficiently high.

111.) Cutoff tests designed to detect purchases made before the end of the year that have been
recorded in the subsequent year most likely would provide assurance about management's assertion
of

a) Presentation and disclosure.

b) Existence or occurrence.

c) Completeness.

d) Valuation or allocation.

112.) Internal control procedures are strengthened when the quantity of merchandise ordered is
omitted from the copy of the purchase order sent to the

a) Purchasing agent.

b) Accounts payable department.

c) Receiving department.

d) Department that initiated the requisition.

**The receiving department copy of the purchase order should be "blind" (i.e., quantities omitted).
This ensures that the receiving department personnel count the incoming merchandise. Thus the
company will end up paying only for what was received, which may not be what was billed by the
vendor.

113.) While observing a client's annual physical inventory, an auditor recorded test counts for several
items and noticed that certain test counts were higher than the recorded quantities in the client's
perpetual records. This situation could be the result of the client's failure to record

a) Sales returns.

b) Sales.

c) Purchase returns.

d) Purchase discounts.
114.) To best ascertain that a company has properly included merchandise that it owns in its ending
inventory, the auditor should review and test the

a) Terms of the open purchase orders.

b) Purchase cut-off procedures.

c) Purchase invoices received on or around year end.

d) Contractual commitments made by the purchasing department.

**Purchase cutoff procedures are designed to determine that items actually received in inventory
have been included in the proper period.

115.) Which of the following would most likely be an internal control procedure designed to detect
errors and irregularities concerning the custody of inventory?

a) Periodic reconciliation of work in process with job cost sheets.

b) Independent comparisons of finished goods records with counts of goods on


hand.

c) Approval of inventory journal entries by the storekeeper.

d) Segregation of the functions between general accounting and cost accounting.

116.) An increased extent of the tests of controls is most likely to occur when:

a) It is a first-time audit.

b) Controls are ineffective and the preliminary control risk assessment is high.

c) Controls are effective and the preliminary control risk assessment is low.

d) The auditor is performing a fraud audit.

**If the auditors determine that the client's internal control is effective at preventing or detecting
misstatements, they will assess the control risk low. They can then accept a higher level of detection
risk, and the substantive testing can be decreased.

117.) Which of the following is a control procedure that most likely could help prevent employee
payroll fraud?

a) The personnel department promptly sends employee termination notices to the


payroll supervisor.

b) Total hours used for determination of gross pay are calculated by the payroll supervisor.

c) Salary rates resulting from new hires are approved by the payroll supervisor
d) Employees who distribute payroll checks forward unclaimed payroll checks to the absent
employees' supervisors.

**Payroll fraud could involve fictitious employees and/or fictitious salary rates. In order to prevent
these frauds, new hires, terminations of employees, and salary rates should be approved by the
personnel department, which in turn should keep the payroll department and employee supervisors
informed on a timely basis.

118.) Transaction authorization within an organization may be either specific or general. An example
of a specific transaction authorization is the

a) Setting of automatic reorder points for material or merchandise.

b) Approval of a detailed construction budget for a warehouse.

c) Establishment of requirements to be met in determining a customer's credit limits.

d) Establishment of sales prices for products to be sold to any customer.

119.) Which of the following internal control activities most likely would prevent direct labor hours from
being charged to manufacturing overhead?

a) Use of time tickets to record actual labor worked on production orders.

b) Periodic independent counts of work in process for comparison to recorded amounts.

c) Comparison of daily journal entries with approved production orders.

d) Reconciliation of work-in-process inventory with periodic cost budgets

**Time tickets should be designed to keep track of hours worked, by whom, and on what production
orders. After approval by a production supervisor, time tickets provide the information on number of
hours worked directly on specific production orders. If time tickets are properly used to record actual
hours worked on orders, approval and/or review of these cards should prevent direct labor hours from
being incorrectly charged to manufacturing overhead.

120.) Which of the following is ordinarily considered a test of controls?

a) Send confirmation letters to banks.

b) Count and list cash on hand.

c) Examine signatures on checks.

d) Obtain or prepare reconciliations of bank accounts as of the balance sheet date.

**Examine signatures on checks is a test of controls; the control is requiring signatures on checks in
place and functioning properly.

121.) An auditor vouched data for a sample of employees in a payroll register to approved clock card
data to provide assurance that
a) Internal controls relating to unclaimed payroll checks are operating effectively.

b) Segregation of duties exist between the preparation and distribution of the payroll.

c) Payments to employees are computed at authorized rates.

d) Employees work the number of hours for which they are paid.

122.) An auditor would consider internal control over a client's payroll procedures to be ineffective if
the payroll department supervisor is responsible for

a) Updating employee earnings records.

b) Hiring subordinate payroll department employees.

c) Applying pay rates to time tickets.

d) Having custody over unclaimed paychecks.

**The payroll accounting department has a recording responsibility and as such it should not have
custody of unclaimed payroll checks (custody of an asset). If the payroll accounting department had
custody of payroll checks, its employees could add a fictitious employee to the payroll and
subsequently obtain the check.

123.) Which of the following procedures in the cash disbursements cycle should not be performed by
the accounts payable department?

a) Canceling supporting documentation after payment.

b) Signing the voucher for payment by an authorized person.

c) Verifying the mathematical accuracy of the vendor's invoice.

d) Comparing the vendor's invoice with the receiving report.

**The supporting documentation should be canceled by the check signer (usually the treasurer) and
not the accounts payable department.

124.) Which of the following tests of controls most likely would help assure an auditor that goods
shipped are properly billed?

a) Compare the accounts receivable ledger to daily sales summaries.

b) Examine shipping documents for matching sales invoices.

c) Inspect unused sales invoices for consecutive prenumbering.

d) Scan the sales journal for sequential and unusual entries.

125.) Which of the following internal control procedures is not usually performed in the vouchers
payable department?
a) Approving vouchers for payment by having an authorized employee sign the vouchers.

b) Indicating the asset and expense accounts to be debited.

c) Matching the vendor's invoice with the related receiving report.

d) Accounting for unused prenumbered purchase orders and receiving reports.

**The unused forms should be accounted for by the department in which the forms are prepared -
purchase orders in the purchasing department and receiving reports in the receiving department.

126.) Internal control over cash receipts is weakened when an employee who receives customer mail
receipts also

a) Prepares bank deposit slips for all mail receipts.

b) Maintains a petty cash fund.

c) Prepares initial cash receipts records.

d) Records credits to individual accounts receivable.

**There is a control weakness because the employee has both custody of an asset (cash) and the
accounting record (accounts receivable subsidiary ledgers).

127.) Which of the following internal control activities most likely would justify a reduced assessed
level of control risk concerning plant and equipment acquisitions?

a) Periodic physical inspection of plant and equipment by the internal audit staff.

b) Comparison of current-year plant and equipment account balances with prior-year actual
balances.

c) The review of prenumbered purchase orders to detect unrecorded trade-ins.

d) Approval of periodic depreciation entries by a supervisor independent of the accounting


department.

**The internal audit procedure of the periodic inspection of physical equipment and comparison to
what is recorded by the internal would allow for a reduction in the scope of the auditor's tests of asset
acquisitions.

128.) A customer buys some furniture from a company and must be billed for the Php879 sales price.
The company’s billings department prepares a sales invoice which is properly approved. Which of
the following does not receive a copy of this sales invoice?

a) The billings department retains a copy for its records.

b) The inventory management department gets a copy to update the perpetual


records.
c) The customer receives a copy.

d) The accounts receivable department is sent a copy.

**The company should have created a shipping document at the time the furniture left the warehouse
to be used to update the perpetual inventory records. The sales invoice is not used for that purpose.
The billings department and accounts receivable should both have a copy and one is mailed to the
customer in hopes of bringing in a cash payment.

129.) The Waynesboro Corporation has recently installed a new computer payroll processing
program. Before the program is used to compute actual payroll checks for the employees, test data
is going to be run through the computer to see how it would be processed. Which of the following is
least likely to be tested in this manner?

a) Two checks requested for the same employee

b) A check requested for an employee who quit two months earlier.

c) A check requested for an employee working more than 60 hours per week.

d) A check requested for an employee paid an hourly rate of $12 per hour.

130.) Risk assessment involves considering threats to the organization’s objectives in the areas of:

a) Operations, financial reporting, and compliance with laws and regulations.

b) Marketing, financial reporting, and compliance.

c) Financial reporting, performance, and marketing.

d) Compliance with laws and regulations, operations, and performance

131.) To obtain an understanding of the entity and its environment, including its internal control, the
auditor should perform each of the following risk assessment procedures except:

a) Inquiries of management and others within the entity.

b) Sampling for variables.

c) Observation and inspection.

d) Analytical procedures.

132.) In accordance with the standards, the second standard of audit fieldwork is expanded from
"internal control" to:

a) The entity and management assertions, including internal control

b) The entity and its environment, including its internal control

c) The entity and its business model, including internal control.


d) The entity, including internal control.

133.) Which of the following is the basic fundamental concept that underlies the audit process?

a.) Skepticism .b.) Materiality. c.) Risk. d.) All of the above.

134.) Management must disclose material weaknesses in internal control:

a) Only if the auditor disclose the weakness as significant.

b) Whenever the weakness is significant to the overall financial reporting objectives.

c) Whenever the weakness is deemed significant to a single class of transactions.

d) If the weakness exist at the end of the year.

**Management must disclose material weaknesses in internal control that exist at year end, since
material weaknesses result in a more than remote likelihood that internal control will not prevent or
detect material financial statement misstatements.

135.) A weakness in internal control over recording retirements of equipment may cause an auditor to

a) Trace additions to the "other assets" account to search for equipment that is still on
hand but no longer being used.

b) Select certain items of equipment from the accounting records and locate them
in the plant.

c) Review the subsidiary ledger to ascertain whether depreciation was taken on each
item of equipment during the year.

d) Inspect certain items of equipment in the plant and trace those items to the
accounting records.

**A weakness in controls over recording equipment retirements increases the risk that equipment
which is removed from the plant is not removed from the accounting records. When the auditor
selects items in the accounting records and tries to locate them in the plant, he may discover retired
equipment that is still in the accounting records.

136.) An effective system of control procedures over the payroll function would include

a) Preparation of payroll transaction journal entries by an employee who reports to the


supervisor of the personnel department.

b) Verification of agreement of job time tickets with employee clock card hours by a
payroll department employee.

c) Custody of rate authorization records by the supervisor of the payroll department.

d) Reconciliation of totals on job time tickets with job reports by employees responsible for
those specific jobs.
137.) For an appropriate segregation of duties, journalizing and posting summary payroll transactions
should be assigned to

a) The treasurer's department.

b) General accounting.

c) Payroll accounting.

d) The internal accounting department.

138.) In a well-designed internal control system, the same employee may be permitted to

a) Mail signed checks, and also cancel supporting documents.

b) Prepare receiving reports, and also approve purchase orders.

c) Approve vouchers for payment, and also have access to unused purchase orders.

d) Mail signed checks, and also prepare bank reconciliations.

**By canceling the supporting documents, the person in authority ensures that the documents are not
presented for payment again.

139.) When considering internal control, an auditor must be aware of the concept of reasonable
assurance which recognizes that

a) The employment of competent personnel provides assurance that the objectives of


internal control will be achieved.

b) The establishment and maintenance of internal control is an important responsibility of


the management and not of the auditor.

c) The cost of internal control should not exceed the benefits expected to be derived
from internal control.

d) The segregation of incompatible functions is necessary to obtain assurance that the


internal control is effective.

140.) Which of the following internal controls most likely would reduce the risk of diversion of
customers’ receipts by an entity's employees?

a) A bank lockbox system.

b) Prenumbered remittance advices.

c) Monthly bank reconciliations.

d) Daily deposit of cash receipts.


**A bank lockbox system assures accountability control as cash enters the client's cash receipts
system.

141.) Reportable conditions are matters that come to an auditor's attention, which should be
communicated to an entity's audit committee because they represent

a) Off-balance sheet transactions that are not fully disclosed in the financial statements.

b) Conditions which may result in a disclaimer of an opinion on the financial statements.

c) Significant deficiencies in the design or operation of the internal control structure

d) Material irregularities or illegal acts perpetrated by high-level management.

142.) Which of the following statements is correct concerning reportable conditions in an audit?

a) Reportable conditions generally lead to an adverse audit opinion.

b) An auditor is required to search for reportable conditions during an audit.

c) All reportable conditions are also considered to be material weaknesses.

d) An auditor may communicate reportable conditions during an audit or after the


audit's completion.

143.) Which of the following conditions is necessary for a practitioner to accept an attest engagement
to examine and report on an entity's internal control structure over financial reporting?

a) Management presents its written assertion about the effectiveness of the internal
control structure.

b) The practitioner is a continuing auditor who previously has audited the entity's financial
statements.

c) The practitioner anticipates relying on the entity's internal control structure in a financial
statement audit.

d) The practitioner is hired by the audit committee to specifically audit internal control in
accordance with generally accepted auditing standards.

**A practitioner may examine and report on management's assertion about the effectiveness of an
entity's internal control structure if the following conditions are met: Management accepts
responsibility for the effectiveness of the entity's internal control structure, management evaluates the
effectiveness of the entity's internal control structure, and presents its written assertion about the
effectiveness of the entity's internal control structure.

144.) Which of the following is an inherent limitation in internal control?

a) Incompatible duties.

b) Lack of segregation of duties.


c) Faulty human judgment.

d) Lack of an audit committee.

145.) Significant deficiencies are matters that come to an auditor's attention and should be
communicated to an entity's audit committee because they represent:

a) Intentional attempts by the client personnel to limit the scope of the auditor's fieldwork.

b) Flagrant violations of the entity's documented conflict-of-interest policies.

c) Material fraud perpetrated by high-level management.

d) Internal control deficiencies that could adversely affect a company's ability to


initiate, record, process, or report external financial statements reliably.

146.) How must significant deficiencies and material weaknesses be communicated to those charged
with governance?

a) Oral communication is required.

b) Written communication is required.

c) Either oral or written communication is acceptable.

d) None of the above is correct.

147.) In determining whether transactions have been recorded, the direction of the auditing testing
should be from the

a) General ledger balances.

b) Adjusted trial balances.

c) Original source document.

d) General journal entries.

148.) Which of the following is the best example of a substantive test?

a) Flowcharting of the client's cash disbursement system.

b) Confirmation of balances of accounts receivable.

c) Examining a sample of cash disbursements to test whether expenses have been


properly approved.

d) Comparison of signatures on checks to an authorized list of signers.

**Confirmation of balances of accounts receivable will provide a test of ending account balance and is
therefore a detail test of a balance, a type of substantive test. Examining approval of cash
disbursements and comparing signatures on checks with authorized signers are examples of tests of
controls; flowcharting is an example of an auditor's documentation of his/her understanding of a
process.

149.) The objective of tests of details of transactions performed as substantive tests is to

a) Evaluate whether management's policies and procedures operated effectively.

b) Attain assurance about the reliability of the accounting system.

c) Detect material misstatements in the financial statements.

d) Evaluate whether management internal control operates effectively.

150.) To determine whether accounts payable are complete, an auditor performs a test to verify that
all merchandise received is recorded. The population of documents for this test consists of all

a) Receiving reports.

b) Vendor's invoices.

c) Purchase orders.

d) Canceled checks.

151.) Which of the following is a substantive test that an auditor most likely would perform to verify
the existence and valuation of recorded accounts payable?

a) Investigating the open purchase file to ascertain that prenumbered purchase orders are
used and accounted for. changes in long-term debt occurring after year end.

b) Receiving the client's mail, unopened, for a reasonable period of time after the year end
to search for unrecorded vendor's invoices.

c) Vouching selected entries in the accounts payable subsidiary ledger to purchase


orders and receiving reports.

d) Confirming accounts payable balances with known suppliers who have zero balances.

152.) Independent auditors have carried out analytical procedures on an audit client and determined
that the reported liabilities at the end of Year One are significantly lower than expected. Which of the
following is most likely to have created this situation?

a) Purchases received during the last few days of Year One were recorded in Year
Two.

b) Sales made during the first few days of Year Two were recorded in Year One.

c) Cash payments made during the last few days of Year One were recorded in Year Two.

d) Cash collections made during the first few days of Year Two were recorded in Year One.
**Sales and cash collections are unlikely to have an impact on the amount of reported liabilities.
However, purchases and cash payments do affect the liability balance. By not recording the
purchases in Year One, the liability was incorrectly omitted and would be too low at the end of the
year. By not recording the cash payments in Year One, the liability (although paid) incorrectly remains
on the books and is too high.

153.) The CPA firm of Terry & Francona is auditing the financial statements of Fenway International
Corporation. Fenway owns a number of unique investments where the market value is difficult to
determine. The CPA firm hires a specialist in investment analysis to help determine the market value
of these assets. Which of the following statements is true?

a) This investment analyst must be independent of the client company.

b) The hiring of this analyst means that the firm must provide a qualified audit opinion.

c) The hiring of this analyst means that the firm must add an extra paragraph to the end of
the audit report to alert readers to the work of the specialist.

d) The firm must assess the qualifications of the specialist as part of the audit
process.

**The CPA firm is required to ensure that the specialist is truly capable of making the assessments
that are being asked. The qualifications must be assessed carefully. If the analyst is not
independent, the CPA firm can still make use of the information provided by the specialist but should
consider carefully the impact of the relationship with the client in making that assessment. Finally,
use of a specialist is a normal audit technique and does not require any change in the audit report.
However, the CPA firm does have the right to make mention that a specialist was used if that
information is viewed as helpful.

154. Auditing standards differ from auditing procedures in that procedures relate to
a) Measure of performance.
b) Audit principles.
c) Acts to be performed.
d) Audit judgments.

155. The independent auditor of 1900 differs from the auditor of today in that the 1900 auditor was
more concerned with the
a) Validity of the income statement.
b) Determination of fair presentation of financial statements.
c) Improvement of accounting systems.
d) Detection of irregularities.

156. The first general standard of generally accepted auditing standards which states, in part, that
the examination is to be performed by a person or persons having adequate technical training,
requires that an auditor have
a) Education and experience in the field of auditing.
b) Ability in the planning and supervision of the audit work.
c) Proficiency in business and financial matters.
d) Knowledge in the areas of financial accounting.

157. The first standard of field work, which states that the work is to be adequately planned, and
assistants, if any, are to be properly supervised, recognizes that
a) Early appointment of the auditor is advantageous to the auditor and the client.
b) Acceptance of an audit engagement after the close of the client's fiscal year is generally not
permissible.
c) Appointment of the auditor subsequent to the physical count of inventories requires a disclaimer
of opinion.
d) Performance of substantial parts of the examination is necessary at interim dates.

158. An independent audit aids in the communication of economic data because the audit
a) Assures the reader of financial statements that any fraudulent activity has been corrected.
b) Confirms the accuracy of management's financial representations.
c) Lends credibility to the financial statements.
d) Guarantees that financial data are fairly presented.

159. A CPA is most likely to refer to one or more of the three general auditing standards in
determining
a) Whether the CPA should undertake an audit engagement.
b) The nature of the CPA's report qualification.
c) The scope of the CPA's auditing procedures.
d) Requirements for the review of internal control.

160. The first standard of field work recognizes that early appointment of the independent auditor
has many advantages to the auditor and the client. Which of the following advantages is least
likely to occur as a result of early appointment of the auditor?
a) The auditor will be able to complete the audit work in less time.
b) The auditor will be able to perform the examination more efficiently.
c) The auditor will be able to better plan for the observation of the physical inventories.
d) The auditor will be able to plan the audit work so that it may be done expeditiously.

161. Which of the following best describes the reason why an independent auditor reports on
financial statements?
a) A management fraud may exist and is more likely to be detected by independent
auditors.
b) Different interests may exist between the company preparing the statements and the persons
using the statements.
c) A misstatement of account balances may exist and is generally corrected as the result of the
independent auditor's work.
d) Poorly designed internal control may exist.

162. What is the general character of the three generally accepted auditing standard classified as
general standards?
a) Criteria for content of the F/S and the auditor's report.
b) Criteria of audit planning and supervision and evidence gathering.
c) The need to maintain an independence in mental attitude in all matters relating to the
assignments.
d) Criteria for competence, independence and professional care of individuals performing
the audit.

163. A CPA, while performing an audit, strives to achieve independence in appearance in order to
a) Reduce risk and liability.
b) Comply with the generally accepted standards of field work.
c) Become independent in fact.
d) Maintain public confidence in the profession.

164. The "generally accepted auditing standards" are standards which


a) Are sufficiently established so that independent auditors generally agree on their existence.
b) Are generally accepted based upon a pronouncement of the Financial Accounting Standards
Board.
c) Are generally accepted in response to the changing needs of the business community.
d) Are generally accepted as a consequence of approval of the AICPA membership.

165. The primary objective of the ordinary examination of financial statement by a CPA is the
expression of an opinion on
a) The competence of management in accounting matters which is implied by whether the opinion
is qualified or not.
b) The conformity of the statements with the book of account.
c) The conformity of the financial statements with generally accepted auditing standards applied
on a basis consistent with that of the prior year.
d) The fairness with which the financial statements present cash flows and results of
operations.

166. The primary responsibility for the adequacy of disclosure in the financial statements of a
publicly held company rests with the
a) Partner assigned to the engagement.
b) Auditor in charge of field work.
c) Management of the company.
d) Securities & Exchange Commission.

167. A CPA should comply with applicable GAAS on every engagement


a) Without exception.
b) Except in examinations that result in a qualified report.
c) Except in engagements where the CPA is associated with unaudited F/S.
d) Except in examinations of interim financial statements.

168. Which of the following best describes what is meant by GAAS?


a) Audit objectives generally determined on audit engagements.
b) Acts to be performed by the auditor.
c) Measures of the quality of the auditor's performance.
d) Procedures to be used to gather evidence to support financial statements.

169. The first general standard recognizes that regardless of how capable an individual may be in
other fields, the individual can not meet the requirements of the auditing standards without the
proper
a) Business and finance course.
b) Quality control and peer review.
c) Education & experience in auditing.
d) Supervision and review skills.

170. The first general standard requires that the examination of F/S is to be performed by a
person having adequate technical training and
a) Independence with respect to the F/S and supplementary disclosures.
b) Exercising professional care as judged by peer reviewers.
c) Proficiency as an auditor which likely has been acquired from previous experience.
d) Objectivity as an auditor as verified by proper supervision.

171. Due professional care requires


a) A critical review of the work done at every level of supervision.
b) The examination of all corroborating evidence available.
c) The exercise of error free judgment.
d) A study and review of the I/C's that include tests of controls

172. The third general standard states that due care is to be exercised in the performance of the
examination. This standard means that a CPA who undertakes an engagement assumes a duty to
perform each audit
a) As a professional possessing the degree of skill commonly possessed by others in the
field.
b) In conformity with generally accepted accounting principles.
c) With reasonable diligence and without fault or error.
d) To the satisfaction of governmental agencies and investors who rely upon the audit.

173. According to court decision, GAAS established by the AICPA applies


a) Only to AICPA members.
b) To all CPA's.
c) Only to those who choose to follow them.
d) Only when conducting audits subject to the AICPA jurisdiction.

174. Which of the following best describes the reason why an independent auditor reports on
financial statements?
a) A management fraud may exist and is more likely to be detected by independent auditors.
b) Different interests may exist between the company preparing the statements and the
persons using the statements.
c) A misstatement of account balances may exist and is generally corrected as the result of the
independent auditor's work.
d) Poorly designed internal control may exist.
(Yes, this is the same as #8, ask me why!!!)

175. The independent audit is important to readers of F/S because it


a) Determines the future stewardship of the management of the company whose financial
statements are audited.
b) Measures and communicates financial and business data in the F/S.
c) Involves the objective examination of, and reporting on, management-prepared
statements.
d) Reports on the accuracy of all information in the F/S.

176. Independent auditing can best be described as


a) A branch of accounting.
b) A discipline that attests to the results of accounting and other functional operations and
data.
c) A professional activity that measures and communicates financial and business data.
d) A regulatory function that prevents the issuance of improper financial information.

177. What is the meaning of the GAAS that requires the auditor to be independent?
a) The auditor must be without bias with respect to the client under audit.
b) The auditor must adopt a critical attitude during the audit.
c) The auditor's sole obligation is to third parties.
d) The auditor may have a direct ownership interest in the client's business if it is not material.

178. The primary purpose of a management advisory services engagement is to help the client
a) Become more profitable by relying upon the CPA's existing personal knowledge about the
client's business.
b) Improve the use of its capabilities and resources to achieve its objectives.
c) Document and quantify its future plans without impairing the CPA's objectivity or allowing the
CPA to assume the role of management.
d) Obtain benefits that are guaranteed implicitly by the CPA.

179. Operational auditing is primarily oriented toward


a) Future improvements to accomplish the goals of management.
b) The accuracy of data reflected in management's financial records.
c) The verification that a company's financial statements are fairly presented.
d) Past protection provided by existing internal control.

180. Because an examination i/a/w GAAS is influenced by the possibility of material errors, the
auditor should conduct the examination with an attitude of
a) Professional responsiveness.
b) Conservative advocacy.
c) Objective judgment.
d) Professional skepticism.

181. The exercise of due professional care requires that an auditor


a) Use error-free judgment.
b) Consider the internal control structure, including tests of controls.
c) Critically review the work done at every level of supervision.
d) Examine all corroborating evidence available.

182. CPA firms should establish quality control policies and procedures for personnel
management in order to provide reasonable assurance that
a) Employees promoted possess the appropriate characteristics to perform competently.
b) Personnel will have the knowledge required to fulfill responsibilities assigned.
c) The extent of supervision and review in a given instance will be appropriate.
d) All of the above are reasons.

183. The least important evidence of a CPA firm's evaluation of its system of QC would concern
the CPA firm's policies and procedures for
a) Employment (hiring).
b) Confidentiality of audit engagements.
c) Assigning personnel to audit engagements.
d) Determination of audit fees.

184. A CPA establishes QC policies and procedures for deciding whether to accept a new client or
continue to perform services for a current client. The primary purpose for establishing such
policies is to
a) Enable the auditor to attest to the integrity or reliability of a client.
b) Comply with the quality control standards established by regulatory bodies.
c) Minimize the likelihood of association with clients whose management lacks integrity.
d) To lessen the exposure to litigation resulting from failure to detect irregularities in client financial
statements.

185. In pursuing its quality control objectives with respect to acceptance of a client, a CPA firm is
not likely to
a) Make inquiries of the proposed client's legal counsel.
b) Review financial statements of the proposed client.
c) Make inquiries of previous auditors.
d) Review the personnel practices of the proposed client.

186. Within the context of quality control, the primary purpose of continuing professional education
and training activities is to enable a CPA firm to provide personnel within the firm with
a) Technical training that assures proficiency as an auditor.
b) Professional education that is required in order to perform with due professional care.
c) Knowledge required to fulfill assigned responsibilities and to progress within the firm.
d) Knowledge required in order to perform a peer review.

187. In pursuing a CPA firms' quality control objectives, a CPA firm may maintain records
indicating which partners or employees of the CPA firm were previously employed by the CPA
firm's clients. Which quality control objective would this be most likely to satisfy?
a) Acceptance of client.
b) Supervision.
c) Independence.
d) Monitoring.

188. In pursuing its quality control objectives with respect to independence, a CPA firm may use
policies and procedures such as
a) Emphasizing independence of mental attitude in firm training programs and in
supervision and review of work.
b) Prohibiting employees from owning stock of public companies.
c) Suggesting that employees conduct their banking transactions with banks that do not maintain
accounts with client firms.
d) Assigning employees who may lack independence to research positions that do not require
participation in field audit work.

189. Which of the following is an element of quality control?


a) Supervision
b) Inspection
c) Personnel management
d) Consultation

190. In connection with the element of monitoring, a CPA firm's system of quality control should
ordinarily provide for the maintenance of
a) A file of minutes of staff meetings.
b) Updated personnel files.
c) Documentation to demonstrate compliance with its policies and procedures.
d) Documentation to demonstrate compliance with peer review directives.

191. One element of the personnel management quality control standard is professional
development. The primary reason why a CPA firm establishes policies and procedures for
professional development of staff accountants is to
a) Comply with the continuing educational requirements imposed by various states for all staff
accountants in CPA firms.
b) Establish, in fact as well as in appearance, that staff accountants are increasing their
knowledge of accounting and auditing matters.
c) Provide a forum for staff accountants to exchange their experiences and views concerning firm
policies and procedures.
d) Provide reasonable assurance that staff personnel will have the knowledge required to
enable them to fulfill responsibilities.

192. Which of the following is a quality control standard?


a) Peer review.
b) Administrative control.
c) Engagement performance.
d) Time studies.

193. What is the responsibility of a successor auditor (SA) with respect to communicating with the
predecessor auditor (PA) in connection with a prospective new client?
a. The SA has no responsibility to contact the PA.
b. The SA should obtain permission from the prospective client to contact the PA.
c. The SA should contact the PA regardless of whether the prospective client authorizes contact.
d. The SA need not contact the PA if the successor is aware of all available relevant facts.

194. A CPA firm's personnel partner periodically studies the CPA firm's personnel advancement
experience to ascertain whether individuals meeting stated criteria are assigned increased
degrees of responsibility. This is evidence of the CPA firm's adherence to prescribed standards of
a) Quality control.
b) Due professional care.
c) Supervision and review.
d) Field work.

195. Quality control for a CPA firm as referred to in Statements on Quality Control Standards,
applies to
a) Auditing services only.
b) Auditing and management advisory services.
c) Auditing and tax services.
d) Auditing and accounting and review services.

196. A prospective client's refusal to grant a CPA permission to communicate with the predecessor
auditor will bear directly on the CPA's ability to
a) Determine appropriate pricing of the audit.
b) Determine the integrity of management.
c) Determine the beginning balances of the current year's financial statements.
d) Establish consistency in application of GAAP between years.

197 . Prior to the acceptance of an audit engagement with a client who has terminated the
services of the predecessor auditor, the CPA should
a. Contact the predecessor auditor without advising the prospective client and request a complete
report of the circumstances leading to the termination with the understanding that all information
disclosed will be kept confidential.
b. Accept the engagement without contacting the predecessor auditor since the CPA can include
audit procedures to verify the reason given by the client for the termination.
c. Not communicate with the predecessor auditor because this would, in effect, be asking the
auditor to violate the confidential relationship between auditor and client.
d. Advise the client of the intention to contact the predecessor auditor and request
permission for the contact.

198. Before accepting an audit engagement, a successor auditor should make specific inquiries of
the predecessor auditor regarding the predecessor's
a. Awareness of the consistency in the application of GAAP between periods.
b. Evaluation of all matters of continuing accounting significance.
c. Opinion of any subsequent events occurring since the predecessor's audit report was issued.
d. Understanding as to the reasons for the change of auditors.

199. The single feature that most clearly distinguishes auditing, attestation, and assurance is
a. Type of service.
b. Training required performing the service.
c. Scope of services.
d. CPA’s approach to the service.

200. The auditor's judgment concerning the overall fairness of the presentation of financial
position, results of operations, and changes in financial position is applied within the framework
of
a. Generally accepted accounting principles.
b. Generally accepted auditing standards.
c. Internal control.
d. Information systems control.

201. The auditor communicates the results of his or her work through the medium of the
a. Engagement letter.
b. Management letter.
c. Audit report.
d. Financial statements.

202. The four major steps in conducting an audit are:


a. Testing internal controls
b. Audit report
c. Planning
d. Testing transactions and balances

The proper sequence in applying the above steps is:


a. cadb b. cdab c. bcda d. adcb

203.) Which of the following best describes the purpose of the engagement letter?
a. The engagement letter relieves the auditor of some responsibility for the exercise of due care.
b. By clearly defining the nature of the engagement, the engagement letter helps to avoid and
resolve misunderstandings between CPA and client regarding the precise nature of the work to
be performed and the type of report to be issued.
c. The engagement letter conveys to management the detailed steps to be applied in the audit
process.
d. The engagement letter should be signed by both the client and the CPA and should be used
only for independent audits.

204. Which of the following is mandatory if the auditor is to comply with generally
accepted auditing standards?
a. Possession by the auditor of adequate technical training.
b. Use of analytical review on audit engagements.
c. Use of statistical sampling whenever feasible on an audit engagement.
d. Confirmation by the auditor of material accounts receivable balances.

205. Which of the following statements best describes why the CPA profession has
deemed it essential to promulgate ethical standards and to establish means for ensuring their
observance?
a. A requirement for a profession is the establishment of ethical standards that stress primarily
a responsibility to clients and colleagues.
b. A requirement of most state laws calls for the profession to establish a code of ethics.
c. An essential means of self-protection for the profession is the establishment of flexible
ethical standards by the profession.
d. A distinguishing mark of a profession is its acceptance of responsibility to the public.

206. Pursuant to the AICPA rules of conduct, the auditor's responsibility to the profession
is defined by
a. The AICPA Code of Professional Conduct.
b. Federal laws governing licensed professionals who are involved in interstate commerce.
c. Statements on Auditing Standards.
d. The Bylaws of the AICPA.

207. The exercise of due professional care requires that an auditor


a. Examine all available corroborating evidence.
b. Critically review the judgment exercised at every level of supervision.
c. Reduce control risk below the maximum.
d. Attain the proper balance of professional experience and formal education.

208. In determining estimates of fees, an auditor may take into account each of the
following, except the
a. Value of the service to the client.
a. Degree of responsibility assumed by undertaking the engagement.
b. Skills required to perform the service.
c. Attainment of specific findings.

Choose the following actions with the Code of Conduct rule violated by the action. No rule is used
more than once. Briefly explain why the action is a violation of the rule cited.

a. Independence
b. Integrity and objectivity
c. General standards
d. Compliance with standards

209. Brianna Lopez, CPA, agreed to review loan applications for First Charter Bank, an
audit client. The bank granted or denied the loans on the basis of Lopez’ recommendations.
A

210. In defense of a lawsuit alleging negligence, Melissa Franklin, CPA, explained that
she was not an expert in commodities trading and therefore did not detect the accounting fraud
perpetrated by her client, a commodities broker.
C

211. In reviewing the corporate tax return for Eager Turnstiles, Inc., Abba Shah, CPA,
discovered that Eager’s controller had incorrectly reported a P500,000 purchase of painting
equipment as repairs expense. When Shah informed the controller of the tax code violation, she
refused to correct the return. Shah signed the return as preparer.
D

212. Ben Williams, CPA, issued an unqualified opinion on a set of financial statements,
even though he felt uncomfortable about an accounting practice applied by the client. Although
the practice in question was in accordance with GAAP, it increased net income significantly
above a level that Williams considered reasonable.
B

Choose the following actions with the Code of Conduct rule violated by the action. No rule is used
more than once. Briefly explain why the action is a violation of the rule cited.

a. Accounting principles
b. Confidential client information
c. Contingent fees
d. Acts discreditable

213. Juanita Garcia, CPA, refused to be associated with a client’s financial statements
after the client declined to correct a material misstatement. Garcia later contacted James Jordan,
CPA, retained by the client to replace Garcia, and informed Jordan of the misstatement.
B

214. Rudy Boesch, CPA, accepted an audit engagement for a fixed fee of P27,000 plus
1% of audited net assets. C

215. Jones Transfer Company wishes to defer charging certain research and
development expenditures to current income on the basis that the expenditures are virtually
certain to benefit future operations. For this reason, Jack Risher, CPA and Jones’ auditor, agrees
with the proposed accounting treatment. A

216. The following are examples of the type of information that may come to the auditor’s
attention which might indicate that noncompliance with laws or regulations has occurred. One of
them is not. Identify the exception:
a. Media comment
b. Industry is regulated by various government agencies
c. Unusual payments in cash, purchases in the form of cashier’s checks payable to bearer or
transfers to numbered bank accounts
d. Payments without proper exchange control documentation

217. As used in PSA 250 (Consideration of Laws and Regulations in an Audit of Financial
Statements), this term refers to acts of omission or commission by the entity being audited, either
intentional or unintentional, which are contrary to prevailing laws or regulations.
a. Noncompliance c. Erotic acts
b. Illegal acts d. Unforgivable acts

218. According to PSA 250, the term “noncompliance” as used in the standards refers to
acts of omission or commission by the entity being audited, either intentional or unintentional,
which are contrary to the prevailing laws or regulations. Such acts do not include
a. Transactions entered into by the entity.
b. Transaction entered into in the name of the entity.
c. Transaction entered into on the entity’s behalf by its management or employees.
d. Personal misconduct (unrelated to the entity’s business activities) by the entity’s management
or employees.

219. The responsibility for the prevention and detection of noncompliance rest with
a. The auditor. c. The auditor’s lawyer.
b. Management d. The client’s lawyer.

220. PSA 250 states that in order to plan the audit, the auditor should obtain a general
understanding of the legal and regulatory framework applicable to the entity and the industry and
how the entity is complying with that framework. To obtain this understanding, the following
procedures would ordinarily be considered by the auditor, except
a. Use the existing understanding of the entity’s industry, regulatory, and other external factors.
b. Inquire of management concerning the entity’s policies and procedures regarding compliance
with laws and regulations.
c. Inquire of management as to the laws and regulations that may be expected to have a
fundamental effect on the operation of the entity.
d. Inspect correspondence with relevant licensing or regulatory authorities.

221. Which of the following statements is incorrect concerning reporting of


noncompliance?
a. The auditors, as soon as practicable, either communicate with those charged with governance,
or obtain evidence that they are appropriately informed, regarding noncompliance that comes
to the auditor’s attention.
b. If the auditor suspects that members of senior management, including members of the board
of directors, are involved in noncompliance, the auditor should report the matter to the next
higher level of authority at the entity, if it exists, such as an audit committee or a supervisory
board.
c. The auditor should, as soon as practicable, communicate with those charged with governance
regarding noncompliance, including matters that are clearly inconsequential or trivial.
d. If in the auditor’s judgment, the noncompliance is believed to be intentional and material, the
auditor should communicate the finding without delay.

222. If the auditor concludes that the noncompliance has a material effect on the financial
statements, and has not been properly reflected in the financial statements, the auditor should
express
a. A qualified or an adverse opinion. c. A disclaimer of opinion.
b. A qualified opinion or a disclaimer of opinion. d. A qualified opinion.

223. If the auditor is precluded by the entity from obtaining sufficient appropriate audit
evidence to evaluate whether noncompliance that may be material to the financial statements, ha,
or is likely to have, the auditor should express
a. A qualified or an adverse opinion. c. An adverse opinion.
b. A qualified opinion or a disclaimer of opinion. d. An adverse opinion or a disclaimer of
opinion.

224. Under which of the circumstances below would the auditor conclude that withdrawal
from the engagement is necessary?
a. The auditor concludes that the noncompliance has a material effect on the financial statements
and has not been properly reflected in the financial statements.
b. The auditor is precluded by the entity from obtaining sufficient appropriate audit evidence to
evaluate whether noncompliance that may be material to the financial statements, has, or is
likely to have, occurred.
c. The auditor is unable to determine whether noncompliance has occurred because of limitations
imposed by the circumstances rather than by the entity.
d. The entity does not take the remedial action that the auditor consider necessary in the
circumstances.

225. Detection of noncompliance, regardless of materiality requires considerations of the


following
a. b. c. d.
Integrity of management Yes Yes Yes No
Possible effect on other aspects of the audit Yes Yes No Yes
Legal determination of the act of non-compliance Yes No Yes No

226. The auditor-in-charge of engagement assesses risk of fraud higher than the
average. The prudent auditor is expected to
a. Assign more experienced auditors to the engagement
b. Assign a more members to the engagement
c. Make a more extensive test of controls
d. Raise the materiality level

227. A type of fraud in which an employee takes assets from an organization for personal
gain.
a. Fraudulent financial reporting
b. Defalcation
c. Window dressing
d. Secret reserve

228. Which of the following is least likely a factor that increases potential for fraud?
a. Operating, financing and investing decisions are dominated by a single person.
b. Operating results are highly sensitive to outside economic conditions.
c. Audit client has been in the business and the leader of the industry for more than a decade.
d. Organization is decentralized without adequate monitoring

229. In assessing potential for fraud, the following are either likely or unlikely to increase
risk of misstatements of financial statements. Which of the following combinations best describes
the likelihood of potential for fraud?
A B C D
Many contentious or difficult accounting issues
are present Likely Likely Likely Unlikely

Significant and unusual related-party transactions


are present Likely Unlikely Unlikely Unlikely
Direction of change in entity’s industry is declining
with many businesses Likely Likely Unlikely Likely

230. Which of the following is an incorrect statement?


a. The amount of audit work should vary inversely with the likelihood of material misstatements
existing in the accounting records.
b. The better the organization’s control structure, the less likely it is that material misstatements
will be present
c. Complex or unusual transactions are more likely to be recorded in error than recurring or
routine transactions are
d. If misstatements are likely to occur in the recording process, the auditor should develop
procedures to detect misstatements.

231. In the regular audit of X Company, B, CPA, discovered a material fraud being
perpetrated by the cashier. What do you expect most of B, CPA to do?
a. Report the incident to the SEC
b. Communicate the existence and details of the fraud to the audit committee of the board of
directors and to management at least one level above that where the fraud has occurred.
c. Advice the shareholders of the client company regarding the fraud.
d. Make an extensive investigation in order to account for the extent of the fraud.

232. How do auditing standards consider the auditor’s responsibilities for detecting fraud
and errors?
a. Auditing standards make no distinction between the auditor’s responsibilities for searching for
errors and fraud.
b. The difficulty of detecting fraud changes the auditor’s responsibility from reasonable to limited
assurance.
c. The auditor should plan to discover material errors and all forms of fraud
d. Auditing standards provide an implicit responsibility for the auditor to discover material
misstatement due to fraud.

233. What should the auditor do first when in an audit of a client entity, an illegal act has
been identified?
a. Consider the effects of the illegal act on the financial statements
b. Communicate the matter with the audit committee of the board of directors
c. Submit a confidential report to the SEC
d. Consult the client’s legal counsel about the matter

234. Lapping is
a. Making the financial statements indicate a more favorable position by giving effect to
tr5ansactions is a period other than that in which these actually occurred
b. Done to inflate the cash position or cover the theft of cash by depositing at the end of the
accounting period a check drawing on one bank account in another bank account without
making the necessary deduction in the balance of the first bank
c. An irregularity that conceals cash shortages by a delay in recording cash collections, retaining
a customer’s payment on credit sales and covering up the shortage with subsequent cash
receipts
d. A kind of fraud committed by making entry of fictitious payments or failure to enter receipts

235. In general, material fraud perpetrated by which of the following are most difficult to
detect
a. Cashier c. Internal auditor
b. Keypunch operator d. Controller

236. Certain management characteristics may heighten the auditor’s concern about the
risk of material misstatements. The characteristic that is least likely to cause concern is that
management
a. Operating and financing decisions are made by numerous individuals
b. Commits to unduly aggressive forecasts
c. Has an excessive interest in increasing the entity’s stock price through use of unduly
aggressive accounting practices
d. In interested in inappropriate methods of minimizing earnings for tax purposes
237. In a financial statements audit, the auditor should consider categories of fraud risk
factors. The auditor is most likely to presume that a high risk of a declaration exists if
a. The client is a multinational company that does business in numerous foreign countries
b. The client does business with several related parties
c. Inadequate segregation of duties places an employee in a position to perpetrate and conceal
thefts
d. Inadequate employee training results in lengthy EDP exception reports each month

238. Which of the following characteristics most likely would heighten an auditor’s
concern about the risk of intentional manipulation of financial statements?
a. Turnover of senior accounting personnel is low
b. Insiders recently purchased additional shares of the entity’s stock
c. Management places substantial emphasis on meeting earnings projection
d. The rate of change in the entity’s industry is slow

239. Under Section 2 of RA 9298, the State recognizes the importance of accountants in
nation building and development. Hence, it shall develop and nurture competent, virtuous,
productive and well rounded professional accountants whose standards of practice and service
shall be excellent, qualitative, world class and globally competitive through
a. Inviolable, honest, effective and credible licensure examinations
b. Regulatory measures, programs and activities that foster their professional growth and
development
a. I only b. II only c. I and II d. Neither I nor II

240. The objectives of the Philippine Accountancy Act of 2004 are the following, except:
a. Standardization and regulation of accounting education.
b. Integration of accountancy profession.
c. Examination for registration of certified public accountants.
d. Supervision, control and regulation of the practice of accountancy.

241. Practice of Public Accountancy shall constitute in a person:


a. When involved in decision making requiring professional knowledge in the science of
accounting, as well as the accounting aspects of finance and taxation.
b. When he/she is appointed in an accounting professional group in government or in a
government-owned and/or controlled corporation, including those performing proprietary
functions, where decision-making requires professional knowledge in the science of
accounting.
c. When he or she is involved in teaching of accounting, auditing, management advisory
services, accounting aspect of finance, business law, taxation and other technically related
subjects.
d. When a person is skilled in the knowledge, science and practice of accounting and as a
qualified person to render professional services as a CPA.

242. Any position in any business or company in the private sector which requires
supervising the recording of financial transactions, preparation of financial statements,
coordinating with the external auditors for the audit of such financial statements and other related
functions shall be occupied only by a duly registered CPA. Provided
a. That the business or company where the above position exists has a paid-up capital of
at least P10,000,000 and/or annual revenue of at least P5,000,000.
b. The above provision shall apply only to persons to be employed after the effectivity of
the Implementing Rules and Regulations of RA 9298.
c. The above provision shall not result to deprivation of the employment of incumbents to
the position.
a. I, II, and III c. II and III
b. I and II d. I and III

243. The following statements relate to the Board of Accountancy. Which statement is
incorrect?
a. The Board consists of a Chairman and six members
b. The Chairman and members are appointed by the President of the Philippines upon
recommendation of PRC
c. No person shall be appointed a member of the Board unless he is natural-born citizen of the
Philippines, a duly registered CPA and has been in the practice of accountancy for at least ten
years
d. The Professional Regulation Commission may remove from the Board any member whose
certificate to practice has been removed or suspended

244. The APO shall submit its nominations with complete documentation to the
Commission not later than ____ prior to the expiry of the term of an incumbent chairman or
member.
a. 30 days b. 60 days c. 90 days d. 120 days

245. Which of the following is incorrect regarding the qualifications of members of the
Board of Accountancy?
a. Must be a natural-born citizen and resident of the Philippines
b. Must be a duly registered Certified Public Accountant with at least fifteen (15) years of work
experience in any scope of practice of accountancy
c. Must be a good moral character and must not have been convicted of crimes involving moral
turpitude
d. Must not have any pecuniary interest, directly or indirectly, in any school, college, university
where review classes in preparation for the licensure examination are being offered or
conducted

246. Which of the following statements is incorrect according to Section 7 of the


Philippine Accountancy Act of 2004 (RA 9298)?
a. The Chairman and the members of the Professional Regulatory Board of Accountancy
(PRBOA) shall hold office for a term of three (3) years.
b. No person who has served two(2) successive terms shall be eligible for reappointment until the
lapse of one (1) year.
c. Any vacancy occurring within the term of a member shall be filled up for the unexpired portion
of the term only.
d. Appointment to fill up an unexpired term is considered as a complete term.

247. The following statements relate to CPA examination ratings. Which statement is
incorrect?
a. To pass the examination, candidates should obtain a general weighted average of 75% and
above, with no rating in any subject less than 65%.
b. Candidates who obtain a rating of 75% and above in at least four subjects shall receive a
conditional credit for the subjects passed.
c. Conditioned candidates shall take an examination in the remaining subject within three years
from the preceding examination.
d. Candidates who fail in two (2) complete CPA examinations may be allowed to take
examinations a third time provided he or she will comply with Sec. 18 of this Act.
248. The Board shall submit to the Commission the ratings obtained by each candidate
within ten (10) calendar days after the examination, unless extended for just cause

Any candidate who fails in two (2) complete Certified Public Accountant Board Examination shall
be disqualified from taking another set of examinations unless he/she submits evidence to the
satisfaction of the Board that he/she enrolled in and completed at least twenty-four (24) units of
subject given in the licensure examination.
a. True, True b. True, False c. False, False d. False, True

249. Which statement is(are) correct regarding CPE requirements for renewal of
professional license?
a. The total CPE credit units required for CPAs shall be sixty (60) units for three (3) years,
provided that a minimum of fifteen (15) credit units shall be earned in each year.
b. Any excess credit units in one year may be carried over to the succeeding years within
the three-year period.
c. Excess credit units earned shall not be carried over to the next three-year period without
exception.
d. A registered professional who is working abroad shall be temporarily exempted from
compliance with CPE requirement during his/her stay abroad, provided that he/she is
has been out of the country for at least one year immediately prior to the date of
renewal.
a. I, II and III only c. I , II and IV only
b. I and II only d. I , II, III and IV

250. Which statement is incorrect regarding CPE requirements for renewal of


professional license?
a. The total CPE credit units required for CPAs shall be sixty (60) units for three (3) years,
provided that a minimum of fifteen (15) credit units shall be earned in each year.
b. A registered professional shall be permanently exempted from CPE requirements upon
reaching the age of 65 years old.
c. A registered professional who is working abroad shall be temporarily exempted from
compliance with CPE requirement during his/her stay abroad, provided that he/she is has been
out of the country for at least one year immediately prior to the date of renewal.
d. Those who failed to renew professional licenses for a period of five (5) continuous years from
initial registration, or from at last renewal shall be declared delinquent.

251. How many credit units per hour is (are) earned by a CPA who serves as a resource
speaker at a CPE seminar?
a. 1 CU per hour c. 3 CU per hour
b. 5 CU per hour d. 2 Cu per hour

252. Generally, the decision to notify parties outside the client’s organization of an illegal
act is the responsibility of the
a. Independent auditor c. Outside legal counsel
b. Management d. Internal auditors

253. An audit should be designed to achieve reasonable assurance of detecting material


a. Errors
b. Errors and irregularities with a direct effect on financial statement amounts
c. Errors, irregularities and those illegal acts with a direct effect on financial statement amounts
and presentation
d. Errors, irregularities and illegal acts

254. Which of the following statements reflects an auditor’s responsibility for detection
fraud and error?
a. An auditor is responsible for detecting employee errors and simple fraud, but not for
discovering fraud involving employee collusion or management override
b. An auditor should plan the audit to detect errors and fraud that are caused by departures from
GAAP
c. An auditor is not responsible for detecting errors and fraud unless the application of GAAS
would result in such detection
d. An auditor should design the audit to provide reasonable assurance of detecting errors and
fraud that are material to the financial statements

255. Which of the following is not an example of an error?


a. Client personnel make mistakes in gathering or processing accounting data from which
financial statements are prepared
b. Client personnel after accounting records from which financial statements are prepared
c. Client personnel overlook or misinterpret facts, causing accounting estimates to be incorrect
d. Client personnel make mistakes in the application of accounting principles

256. Under PSA 260, this term is used to describe the role of persons entrusted with the
supervision, control, and direction of an entity.
A. Oversight C. Direction
B. Governance D. Control

257. According to PSA 260, those matters that arise from the audit of financial statements
and, in the opinion of the auditor, are both important and relevant to those charged with
governance in overseeing the financial reporting and disclosure process are called
A. Audit matters of governance interest. C. Auditor’s findings.
B. Significant audit matters. D. Material misstatements in the financial
statements.

258. Which of the following statements relating to communication of audit matter of


governance interest is incorrect?
A. Audit matters of governance interest include only those matters that have come to the attention
of the auditor as a result of the performance of the audit.
B. In an audit in accordance with PSAs, the auditors should design audit procedures for the
specific purpose of identifying matters of governance interest.
C. The auditor should identify relevant persons who are charged with governance and with whom
audit matters of governance interest are to be communicated.
D. The auditor’s communications with those charged with governance may be made orally or in
writing.

259. While performing professional services for their clients, CPAs have always had a duty to
exercise a level of care which is best described as:
a. Greater than average c. Infallible
b. Superior d. Reasonable

260. S1: Integrity is attribute that most clearly differentiates a CPA who audits management’s
financial statements as contrasted to management.
S2. Not all engagements require integrity
a. True, True c. False, true
b. True, false d. False, false

261. Karen Corporation has engaged Gelai, CPA, to issue a report on the accuracy of product
quality specifications included in trade sales agreements. This is an example of a(n):
a. Attestation service. c. Compliance audit.
b. Financial statement audit. d. Operational audit.

262. One of the general principles of an audit is compliance with Philippine Standards on Auditing
(PSAs). As a consequence of his failure to adhere to PSAs in the course of his examination of the
Mariz Combined Machinery, Inc., Eric, CPA, did not detect the embezzlement of a material amount
of funds by Ubebe Gandarita, the company’s controller. As a matter of common law, to what extent
would Eric be liable to Mariz Combined Machinery, Inc., for losses attributable to the theft?
a. Eric would be liable for losses attributable to his negligence.
b. Eric would be liable only if it could be proven that he committed gross negligence.
c. Eric would have no liability because privity of contract is lacking.
d. Eric would have no liability, since the ordinary examination cannot be relied upon to detect
embezzlement.

263. The following, except one, is always present in attest engagements. Select the exception:
a. The issuance of a written report.
b. The consideration of internal control.
c. The presence of written assertions which is the responsibility of another party.
d. Independence of mind and in appearance on part of the auditor.

264. A technique for regularly and systematically appraising a unit of function and its effectiveness
against corporate and industry standards with the objective of assuring management that its aims
are being carried out and/or identifying conditions that provide an opportunity for improvement:
a. Financial audit c. Compliance audit
b. Operational audit d. management audit

265. Lotlot, CPA, has issued a report with the following wordings: “In our opinion, the schedule
presents, in all material respects…”. In this situation, Lotlot’s report is likely to be:
a. A standard audit report.
b. An audit report with a qualified opinion.
c. An attestation with a qualified opinion.
d. An attestation report.

266. An audit should be designed to achieve reasonable assurance of detecting material:


a. Errors.
b. Errors and irregularities.
c. Errors, irregularities, and those illegal acts with a direct effect on financial statement amounts.
d. Errors, irregularities and illegal acts, regardless of whether it has a direct or indirect effect on
financial statement amounts.

267. Evidence gathering in a scientific experiment is most similar to which phase of the audit?
a. Planning and supervising an engagement.
b. Hypothesizing that financial statements are present fairly.
c. Evaluating whether the financial statements are fairly presented.
d. Assessing risks and designing audit procedures to address these risks.

268. The review of a company’s financial statements by a CPA firm:


a. Culminates in issuance of a report expressing the CPA’s opinion as to the fairness of the
statements.
b. Is substantially less in scope of procedures than an audit.
c. Is of similar scope as an audit and adds similar credibility to the statements.
d. Requires detailed analysis of the major accounts.

269. Berto, CPA, discovered an illegal act during the audit of UBE Corporation, a publicly held
company. Accordingly, which of the following would be the best response by Berto?
a. Berto shall notify the Securities and Exchange Commission regarding the illegal act.
b. Berto should determine who was responsible for the illegal act.
c. Berto shall report the act to the audit committee and high level personnel within Ube
Corporation.
d. Berto shall intensity the examination to identify all illegal acts.

270. In determining estimates of fees, an auditor may take account each of the following, except
the:
a. Value of the service to the client.
b. Degree of responsibility assumed by undertaking the engagement.
c. Skills required to perform the service.
d. Attainment of specific findings.

271. Cruz, Del Rosario, CPAs, was hired by Ariel Rivera Company to give an opinion on recently
prepared, but still unaudited, financial statements. The statement that best describes this
engagement is:
a. The CPA is performing an examination of the financial statements rather than an accounting
service.
b. The financial statements are representations of both management and the CPA.
c. The CPA is performing and accounting service rather than an examination of the financial
statements.
d. The CPA may prepare the statements from the books, but may not assist in adjusting and
closing the books.

272. During planning, one of the auditor’s considerations is the presence of fraud risk factors. Which
of the following factors would most likely heighten an auditor’s concern about the risk of fraudulent
financial reporting?
a. Presence of large amounts of liquid assets that are readily convertible into cash.
b. Low growth and profitability ratios as compared to other entities within the same industry.
c. An overly complex organizational structure involving unusual lines of authority.
d. Top management’s admission of responsibility for the establishment and maintenance of
internal controls.

273. The following are required audit procedures in every audit engagement:
A b c d
Risk assessment procedures Yes Yes Yes Yes
Tests of controls Yes No Yes No
Substantive tests Yes Yes No No

274. Which of the following statements is not correct?


a. Objectively requires that internal auditors have an independent mental attitude.
b. Internal auditors should be independent of the activities they audit.
c. It is acceptable for internal auditors to recommend changes in operations and to install and
implement the operating systems, as long as they do not have the responsibility for operating
them throughout the year.
d. The internal auditor should not be responsible for correcting deficiencies when ineffective or
inefficient operations are found.

275. Which of the following phrases is generally included in an audit engagement letter?
a. “The following are factors considered in setting a preliminary judgment about materiality…”
b. “These are the procedures that the audit team intends to undertake…”
c. “The auditors are responsible to search for significant deficiencies in internal control…”
d. “Company management is responsible for the entity’s compliance with applicable laws, rules
and regulations…"

276. The financial reporting framework adopted by management in preparing the financial
statements that the auditor has determined is acceptable in view of the nature of the entity and the
objective of the financial statements, or that is required by law or regulations.
a. Applicable financial reporting framework.
b. Generally accepted accounting principles.
c. Philippine standards on auditing.
d. Other comprehensive basis of accounting.

277. Which of the following factors most likely would influence an auditor’s determination of the
auditability of an entity’s financial statements?
a. The adequacy of the underlying accounting records.
b. The existence of related parties and related party transactions.
c. The complexity of the accounting system.
d. The operating effectiveness of control procedures and the control environment.

278. Which of the following factors is least likely considered when an auditor is performing
acceptance/continuance procedures?
a. The auditor’s independence of mind and in appearance.
b. Professional competence and ability to provide services to the client.
c. Ability of the client to achieve an unqualified opinion.
d. Integrity of the client’s management and those charged with governance.

279. In an audit in accordance with generally accepted auditing standards, the auditor’s must test
compliance with those laws and regulations that have:
a. A direct and material effect on the financial statements.
b. A direct and material effect on major government programs.
c. A material direct or indirect effect on the financial statements.
d. A material effect on major or non-major audit programs.

280. Professional experience is an important aspect of the training and proficiency of the junior
assistant just entering upon an auditing career. Professional experience should be obtained:
a. Through a thorough study of the generally accepted auditing standards.
b. With proper supervision and review of work by a more experienced supervisor.
c. By completing a number of continuing professional education courses each year.
d. By taking appropriate professional certification exams.

281. The following matters, would be considered by the auditor in planning for an audit of financial
statements (select the exception):
a. Preliminary judgment about materiality levels for audit purposes.
b. Anticipated reliance in internal controls.
c. Financial statement items that possess great risk of material misstatement.
d. The kind of opinion (unqualified, qualified, or adverse) or disclaimer of opinion, likely to be
given.

282. An audit program is usually prepared after establishing the audit strategy and developing the
audit plan. The procedures specifically outlined in an audit program are primarily designed to:
a. Test all material transactions.
b. Gather corroborating evidence.
c. Serve as protection of the auditor in case of litigation regarding negligence.
d. Detect errors or irregularities.

283. Which of the following circumstances most likely would cause an auditor to believe that
material misstatements may exist in an entity’s financial statements?
a. ABC Company told its auditor that “audit trails of computer-generated transactions exist only
for a short time”.
b. Jeff Hong, chief financial officer of DEF Manufacturing, does not want to sign the management
representation letter until the last day of the auditor’s field work.
c. The management of XYZ Enterprises consults with other accountants about significant
accounting matters.
d. The accounts receivable of Jess Inc. were circularized, and the procedure yielded significantly
fewer responses than expected.

284. By common agreement, the audit team assigned to Irah Mae Company decided to set the
preliminary judgment about materiality at P100,000 or 10% of total assets. In addition, the team
decided to allocate the P100,000 to the balance sheet accounts. How will the allocation be
implemented?
a. Allocate the P100,000 based on the relative carrying values of each balance sheet account.
b. Allocate the P100,000 based on professional judgment.
c. Allocate the P100,000 according to the specific formula required in PSA 320, Audit materiality.
d. 10% of the account or P100,000, whichever is lower.

285. An item is considered material to a company’s financial statements if the item:


a. Exceeds 5% of reported income from operations before interest and income taxes, or P10,000,
whichever is lower.
b. Causes the company’s statement of comprehensive income to fall short of the expectations of
external financial analysis.
c. In substance reflects a new class of transactions or events during the period under audit.
d. Alters the total mix of information significantly.

286. Considering the work of experts is one of the factors of audit planning. Which of the following
statements is incorrect concerning the use of the work of experts?
a. An expert under employ by the client cannot be used for purposes of the auditor’s work
because doing so would the rules on objectivity and independence.
b. An auditor may use an expert in the determination of physical characteristics relating to
inventories.
c. In the context of PSAs, an expert is a person possessing a high degree of skill or knowledge in
fields outside auditing and accounting.
d. Professional competence and reputation are some of the major considerations in selecting an
expert.
287. The auditor faces a risk that the examination will not detect material misstatements in the
financial statements. In regard to minimizing this risk, the auditor primarily relies on:
a. Substantive tests. c. Internal control.
b. Tests of controls. d. Statistical analysis.

288. Early appointment of the independent auditor will enable:


a. A more thorough examination to be performed.
b. A proper study and evaluation of internal control to be performed.
c. Sufficient, appropriate evidential matter to be obtained.
d. A more efficient examination to be planned.

289. In an audit situation, communication between the successor and predecessor auditors should
be:
a. Authorized in an engagement letter.
b. Acknowledged in a representation letter.
c. Either written or oral.
d. Written and included in the working papers.

290. A professional accountant who does not consider and apply the guidance included in a
relevant Practice Statement should be prepared to:
a. Explain to the Board of Accountancy tribunal why the practice statement was not complied
with.
b. Bear the consequences of such non-compliance, such as suspension of revocation of license,
plus imprisonment of not more than two (2) years.
c. Face the shareholders of the entity, and explain in the meeting why such practice statement
was not complied with, provided that the explanation is also put in writing, and signed in the
presence of the Chairperson of the Auditing and Assurance Council.
d. Explain how the basic principles and essential procedures in the Engagement Standards
addressed by the Practice Statements have been complied with.

291. Which of the following documentation is required for an audit in accordance with generally
accepted auditing standards?
a. An internal control questionnaire.
b. A client engagement letter.
c. A planning memorandum or checklist.
d. A client representation letter.

292. Which statement is correct relating to a potential successor auditor’s responsibility for
communicating with the predecessor auditors in connection with a prospective new audit client?
a. The successor auditors have no responsibility to contact the predecessor auditors.
b. The predecessor auditors should obtain permission from the prospective client to contact the
successor auditors.
c. The successor auditors should contact the predecessor regardless of whether the prospective
client authorizes contact.
d. Refusal by the prospective client to authorize communication between successor auditor and
predecessor auditor may lead the successor auditor to question the integrity of the prospective
client’s management.

293. An external auditor is determining how the client’s internal auditor could provide direct
assistance in the engagement. Accordingly, the internal auditor may provide direct assistance to
the external auditor in:
a. Obtaining an understanding of internal control systems
b. Performing tests of controls
c. Performing substantive controls
d. All of the answers

294. An engagement in which an auditor is engaged to carry out those procedures of an audit
nature to which the auditor and the entity and any appropriate third parties have agreed and to
report on factual findings.
a. Special audit engagement.
b. Agreed-upon procedures engagement.
c. Examination of prospective information.
d. Engagement to compile information.

295. An increase in the reliance on substantive tests usually mean that the reliance placed on
internal controls:
a. Also increases c. Remains the same
b. Decreases d. Cannot be determined

296. Which of the following are considered further audit procedures that may be designed after
assessing the risks of material misstatement?
a b c d
Tests of controls Yes No Yes No
Substantive tests Yes Yes No No

297. The susceptibility of an account balance to error that could be material, assuming there are no
related controls, is referred to as:
a. Intangible risk c. Inherent risk
b. Detection risk d. Control risk

298. Which of the following ultimately determines the specific audit procedures necessary to provide
an independent auditor with a reasonable basis for the expression of an opinion?
a. The audit program. c. GAAS.
b. The auditor’s judgment. d. The audit documentation.

299. An auditor should obtain sufficient knowledge of an entity’s information system to understand
the:
a. Process used to prepare significant accounting estimates.
b. Controls used to assure proper authorization of transactions.
c. Safeguards used to limit access to IT facilities.
d. Controls used to detect the concealment of fraud.

300. An auditor evaluates the existing system of internal control in order to:
i. Determine the extent of substantive tests which must be performed
ii. Determine the extent of compliance tests which must be performed
iii. Ascertain whether irregularities are probable
iv. Ascertain whether any employees have incompatible functions

301. This refers to a professional who acts as discussion leader or lecturer in a convention or
seminar or similar gathering:
i. Resource speaker c. CPE provider
ii. Peer reviewer d. Panelist/reactor
302. The following are the more significant powers and functions of the Board of Accountancy select
the exception)
a. To prescribe and adopt the rules and regulations necessary for carrying out the provisions of
RA 9298.
b. To supervise the registration, licensure and practice of professions in the Philippines
c. To administer oaths in connection with the administration of RA 9298
d. To revoke of licenses of violators of RA 9298

303. Which of the following best describes the most important stage of an auditor’s statistical
analysis of significant ratios and trends?
a. Computation of significant ratios and trends
b. Interpretation of significant variations and unusual relationships
c. Reconciliation of statistical data to the client’s accounting records
d. Comparison of statistical data to prior year statistics and to similar data published by
governmental and private sources.

304. The probability that an auditor will give an inappropriate opinion on financial statements is:
a. Audit risk c. Control risk
b. Inherent risk d. Detection risk

305. One of the factors affecting the control environment is management philosophy and operating
style. This factor most likely would have a significant influence on an entity’s control environment
when:
a. Management is dominated by one individual
b. Accurate management job descriptions delineate specific duties
c. The audit committee actively oversees the financial reporting process
d. The internal auditor reports directly to management

306. Which of the following would be of least interest to the auditors on considering internal control?
a. Procedures that are concerned with the decisio9n processes leading to management’s
authorization of transactions
b. Procedures restricting access to assets
c. Procedures related to recording transactions
d. Policies concerning the reconciliation of accounting records to existing assets

307. As used in Annex C of the implementing Rules and Regulations to RA 9298, this refers to a
guarantee degree in accountancy, business, or related field from a recognized school, college or
university.
a. Masteral degree c. BSA
b. Doctoral degree d. MSA

308. This refers to the ownership of intellectual property which includes technical or professional
books, instructional materials and the like
a. Authorship c. Patent
b. Copyright d. Professional journalism

309. Ten credit hours as participant of a Continuing Professional Education (CPE) program, activity
or source shall be equivalent /to:
a. 10 credit units c. 3 credit units
b. 5 credit units d. 1 credit units
310. Which of the following is an advantage of describing internal control through the use of a
standardized questionnaire?
a. Questionnaires highlight weakness in the system
b. Questionnaires are more flexible than other methods of describing internal control
c. Questionnaires usually identify situations in which internal control weaknesses are
compensated for by other strengths in the system
d. Questionnaires provide a clearer, more specific portrayal of a client’s system than other
methods of describing internal control

311. An auditor generally obtains evidence regarding the segregation of duties related to inventory
by:
a. Test counts and cur-off procedures
b. Analytical procedures and invoice recomputation
c. Document inspection and reconciliation
d. Personal inquiry and observation

312. A well designed system of internal control that is functioning effectively is most likely to detect
an irregularity arising from:
a. The fraudulent action of several employees
b. The fraudulent action of an individual employee
c. Information deviations from the official organization chart
d. Management fraud

313. The most6 important aspect of any system of controls is:


a. Proper authorization procedures
b. Competent, trustworthy personnel
c. Separation of duties
d. Regular review by top management

314. Obtaining an understanding of an entity’s internal control, an auditor is required to obtain


knowledge about the:
a. b. c. d.
Operating effectiveness of controls Yes No Yes No
Design and operation of controls Yes Yes No No

315. In a small Company that employs inadequate number of employees to permit proper division of
responsibilities, effective internal control can best be strengthen by:
a. Delegation of full clear cut responsibility to each employee for the functions assigned to each
b. Employment of temporary personnel to aid in the separation of duties
c. Direct participation by the owner of the business in the record keeping activities of the
business
d. Affirm in writing management’s approval of limitation on the scope of the audit

316. The following statements relates to RA 9298. Which statement is correct?


a. The Professional Regulation Commission has the authority to remove after due process, any
member of the Board of Accountancy for negligence, incompetence or any other just cause
b. The functions of the Board of Accountancy include the preparation of the content of the CPA
licensure examination but not the rating of examination papers
c. After two years, subject to certain conditions, the Board of Accountancy must order the
reinstatement of a CPA whose certificate of registration has been revoked.
d. Insanity is ground for issuing a certificate of registration to a successful CPA candidate
317. The principal purpose in conducting a study and evaluation of existing internal control system
is
a. For independent auditor to maintain a statement of independence in mental attitude in all
matters relating the audit
b. To assure compliance with GAAP
c. To enable independent auditor to assess and be assured of management’s efficiency and
effectivity
d. To develop an audit plan and determine the nature, timing and extent of audit work required

318. Procedures directed toward obtaining evidential matter concerning the effectiveness of the
deign or operation of an internal control structure policy or procedures are referred to as:
a. Compliance tests
b. Substantive tests
c. Tests of controls
d. Test of internal control structure elements

319. S1 The basic components operations and the primary subject matter of internal accounting
control are transactions
S2 Narratives, flowcharts, and internal control questionnaires are three commonly used methods
for documenting the auditor’s understanding of the internal control structure.
Which of the above statements is correct?
a. S1 only c. Both S1 and S2
b. S2 only d. Neither are correct

320. Which of the following parts of the audit is described by this statement? “the auditor examines
and evaluates processes that produce the numbers and disclosures in the financial statements.”
a. Planning
b. Studying and testing internal control
c. Performing substantial tests
d. Issuing the audit report

321. Financial statement assertions are established for:


a. b. c. d.
Account balances No Yes Yes Yes
Classes of transactions Yes Yes Yes No
Disclosures Yes No Yes No

322. Which of the following should be the focus of analytical procedures in the context of planning
an audit engagement?
a. Enhancing the auditor’s understanding of the entity and its environment
b. Providing assurance that potential material misstatements will be identified
c. Reducing the scope of tests of controls and substantive tests
d. Assessing the sufficiency and appropriateness of audit evidence

323. There are three categories of financial statement assertions: Assertions pertaining to account
balances at period end, assertions pertaining to classes of transactions and events during the
period, and assertions pertaining to presentation and disclosure. Which of the following is a
financial statement assertion that is common to all three categories?
a. Existence c. Completeness
b. Classifications d. Occurrence
324. What type of analytical procedure would an auditor most likely use in developing relationships
among balance sheet accounts when reviewing the financial statements of a non public entity?
a. Trend analysis c. Ratio analysis
b. Regression analysis d. Risk analysis

325. Analytical procedures are required in which phases of the audit engagement?
a. b. c. d.
Planning No Yes Yes Yes
Substantive testing Yes Yes Yes No
Audit completion Yes No Yes Yes

326. In auditing accounts receivable and accounts payable, the auditor’s procedures most likely
would focus primarily on which of the following assertions?
a. Completeness (AR) and existence (AP)
b. Existence (AR) and completeness (AP)
c. Cut-off and occurrence (AR and AP)
d. Existence and occurrence (AR and AP)

327. Philippine Auditing Practice Statements (PAPSs):


a. Are intended to replace Philippine Standards on Auditing (PSAs)
b. Are intended to have the same authority as the PSAs
c. Are issued to provide practical assistance to auditors in implementing PSAs.
d. Are interpretations of PSAs as issued by the Auditing and Assurance Standards Council

328. The following statements relate to RA 9298 and its IRR. Which statement is incorrect?
a. A person shall be considered to be in the professional practice of accounting if, as an officer of
a private enterprise, he makes decisions requiring professional accounting knowledge
b. When a CPA represents his/her client before government agencies o tax and other matters
related to accounting, the CPA is engaged in public accountancy
c. A registered professional shall be permanently exempted from CPE requirements upon
reaching the age of 65 years old
d. The total CPE credit units required for CPAs shall be sixty (60) units for three years, provided
that a maximum of fifteen (15) credit units shall be earned in each year

329. Which of the following is not an underlying condition that creates a demand by users for
reliable information?
a. Transactions that are numerous and complex
b. Decisions are not time sensitive
c. Financial decisions that are important to investors and users
d. Users separated from accounting records by distance and time

330. During the performance of risk assessment procedures, Andoy, CPA, noted a change in
accounting principle has been effected by the client. In this case, Andoy should plan to evaluate
the change to satisfy himself that:
a. The newly adopted principle is a generally accepted accounting principle
b. The method of accounting for the effects of the change is in conformity with generally accepted
accounting principles
c. Management’s justification for the change is reasonable
d. All of the answers

331. In a review engagement the public accountant provides negative assurance in the report which
is issued. This can best be described by the phrase:
a. Nothing has come to my attention
b. The financial statements present fairly
c. We are unable to express an opinion
d. Except for the issues explained above

332. Gina Dizon, CPA, is performing pre-engagement activities. Which of the following must Gina
obtain from her predecessor, prior to the acceptance of the engagement?
a. Analysis of income statement accounts
b. Analysis of balance sheet accounts
c. Facts that might bear on the integrity of management
d. All matters of continuing accounting significance

333. Which of the following best describes due care?


a. Tact in avoiding legal liability c. Reasonable infallibility
b. Requisite skill and diligence d. Freedom from undue influence

334. The audit risk model consists of: AR = IR X CR X DR

The detection risk is the dependent variable. What is the acceptable level of detection risk if the
assessed level of Inherent Risk is High and the Control risk is Low?
a. Highest b. Lower c. Medium d. Higher

335. This refers to the audit procedures deemed necessary in the circumstances to achieve the
objective of the review
a. Scope of an audit c. Audit program
b. Scope of a review d. Scope limitation

336. A certificate under seal, issued by the Commission upon the recommendation by the Board of
Accountancy pursuant to the revised IRR, attesting that individual CPAs (including the staff
members thereof), firms (including the sole proprietors and the staff members thereof) and
partnerships of CPAs (including the partners and the staff members thereof) are duly accredited to
practice public accountancy in the Philippines
a. Certificate of registration c. Certificate of Identification
b. Certificate of accreditation d. Certificate of Quality Review

337. If the auditor concludes that there is reasonable justification to change the engagement and if
the audit work performed complies with the PSAs applicable to the changed engagement, the
report issued would b e that appropriate for:
a. The original engagement, without reference to the original engagement
b. The revised terms of engagement, without reference to the original engagement
c. The revised terms of engagement, with reference to the original engagement
d. The original engagement, with reference to the original engagement

338. Which of the following services provides the highest level of assurance to third parties about a
company’s financial statements?
a. Audits b. Reviews c. Compilation d. Write-up work

Potrebbero piacerti anche